Zuku Review

Ace your homework & exams now with Quizwiz!

A 16-year-old retired Thoroughbred mare is presented for spring vaccinations. Her feet look like this (see image below). Which one of the following would be recommended? A - Recommend 24/7 pasture turnout with grazing B - Ask if she's being fed a diet with excessive wheat bran, leading to an imbalance in the calcium:phosphorus ratio C - Measure resting ACTH and insulin D - Check serum vitamin E and selenium levels E - Start on a course of systemic steroid therapy

Measure resting ACTH and insulin These hooves show evidence of chronic laminitis (rings) One of the most common causes of chronic laminitis is equine metabolic syndrome (EMS), characterized by insulin dyes regulation, regional or generalized adiposity, and laminitis. Grazing is typically contraindicated in horse with EMS because the frusta s in fresh grass exacerbate the insulin response and laminitis. Steroids also worsen insulin Sys regulation and should be avoided in horses with EMS.

When a sow is giving birth, what is the shortest time interval between passage of piglets that indicates dystocia? A - 1 hour B - 15 minutes C - 2 hours D - 1/2 hour E - 4 hours

1 hour Dystocia in the sow is present when one hour or more has passed between piglets. Intervention should be swift to prevent death of piglets from anoxia. In sows, dystocia is often due to uterine inertia, similar to other animals that produce multiple offspring. Other causes include fetal malposition, Fetopelvic disproportion, obstruction of the pelvic canal, deviation of the uterus, and excitement in the dam. Medical therapy can be given with oxytocin and/or calcium only when an obstructive dystocia is not present.

What is the earliest stage post-breeding that an experienced practitioner can reliably rectally palpate fremitus in the uterine artery of the gravid uterine horn in a pregnant cow? A - 45-60 days B - 60-75 days C - 70-80 days D - 120-150 days E - 90 -120 days

120-150 days Fremitus = vibration in the uterine artery of the gravid uterine horn. Remember that fremitus in the uterine artery is a supportive sign of pregnancy, but not conclusive evidence. An experienced practitioner can reliably feel the membrane slip of chorioallantoic membranes rectally at 30-35 days and can feel placentomes at around 70-75 days. Placentomes and membrane slip ARE cardinal signs of pregnancy. One way to remember these is that earliest occurs in reverse alphabetical order. Slip 30-35 days Placentomes 70-75 days Fremitus 120-150 days

At what age do a cow's permanent 4th incisors begin to erupt in the mandible? A - 2 to 2.5 years B - 3 years C - 5-6 years D - 7-8 years E - 3.5 to 4 years

3.5 - 4 years The 4th incisors are actually canine teeth. Cattle age is not always given in a history and estimating age during physical exam helps narrow differentials. References for times of eruption vary by about six months because the times of tooth eruption vary among individual animals and breeds. Full eruption of a new tooth may take another six months. Remember: ruminants don't have upper incisors but instead have a dental pad. Merck Manual tooth eruption reference: https://www.merckvetmanual.com/digestive-system/dental-development-and-anatomy/estimation-of-age-by-examination-of-the-teeth-in-animals

Which of the following is the correct location for placement of a proximal paravertebral block used to perform a standing laparotomy in a cow? A - In the spaces between L1, L2, L3, and L4 B - Above and below the transverse processes of L1, L2 and L4 C - At the ends of the transverse processes of L1, L2, and L4 D - Midway between spine and the ends of the transverse processes of L1, L2, L3 E - Caudal to the transverse processes of T13, L1, and L2

A proximal paravertebral block is performed via placement of local anesthetic just off the midline and caudal to the transverse processes of T13, L1, and L2 vertebrae. With a distal paravertebral block, local is placed above and below the ends of the transverse processes of L1, L2, and L4 vertebrae. The spinal nerves, T13, L1 and L2 are targeted by a paravertebral block to completely desensitize the flank of a cow. Both paravertebral blocks will affect the same nerves because they gradually course caudally after exiting their spinal foramen. Proper placement results in warming of the skin from vasodilation, anesthesia of the skin and body wall, and a curvature of the spine in some cows. The latter is caused by relaxation of the epaxial musculature on the affected side; the spine curves towards the opposite side. Local anesthesia & analgesia: https://www.westernu.edu/mediafiles/veterinary/vet-anesthesia-analgesia/local-anesthesia-analgesia.pdf

A ten-year-old Arabian gelding in California is presented because of an interesting finding in his manure. The owner mentions that he's had a couple of mild colic episodes over the past months that resolved with just walking and oral flunixin meglumine administered by the owner. The manure is shown below: What diagnostic should be recommended to the owner to determine how to proceed? A - Fecal occult blood test B - Abdominal radiographs C - McMasters fecal flotation on fresh manure D - Gastroscopy and duodenal biopsy E - Ultrasound per rectum

Abdominal radiographs There is an enterolith in the manure so abdominal radiographs should be recommended. There is often more than one, and abdominal radiographs are the least invasive way to evaluate for more (80% sensitivity) Exploratory laparotomy is the definitive method but is invasive, costly, and not warranted without radiographs and/or clinical signs. Enteroliths are composed of magnesium, ammonium, and phosphate and typically form around a industrial of sand or foreign material. They are most common in Arabians that eat alfalfa hay in certain regions of the USA.While small stones can be passed, enteroliths more often become too large to pass per rectum. They cause recurrent colic or acute colic when they become lodged in narrow bowel. Definitive Tx - ventral midline cello Tommy to remove the stones via enterotomy. Prognosis: excellent

A commercial pig farm experiences an explosive outbreak of respiratory disease with high mortality, primarily in young pigs under six months of age. Affected pigs show severe respiratory distress, fever up to 107°F (41.5°C), anorexia, and reluctance to move. Some animals display open-mouth breathing with a blood-stained, frothy nasal and oral discharge. On necropsy, the lungs are bilaterally dark and swollen and ooze bloody fluid from the cut surface. Which one of the following choices is the most likely diagnosis? A - Actinobacillus pleuropneumoniae B - Mycoplasma hyopneumoniae C - Haemophilus parahaemolyticus D - Swine influenza E - Fusobacterium necrophorum

Actinobacillus pleuropneumoniae This is a classic scenario of an outbreak of pleuropneumoniae, caused by Actinobacillus pleuropneumoniae. Primarily a growing pig disease, although adults can be affected as well, and sows can abort. Mortality is high if untreated. Survivors may experience lowered growth rates and have a persistent cough.

What kind of organism causes equine granulocytic anaplasmosis? A - Protozoa B - Spirochete C - Ehrlichia D - Chlamydia E - Anaplasma

Anaplasma The causative organism, anaplasma phagocytophilum, used to be classified as ehrlichia but is now classified as an anaplasma. Do not confuse equine anaplasmosis, a necrotizing vasculitis, with BOVINE anaplasmosis, which primarily causes an anemia with icterus and fever.

Which type of immune reaction is occurring in this picture of a horse with limb edema secondary to a localized vasculitis? A - Cell-mediated immune reactions (Type IV) B - Immediate hypersensitivity and atopy (Type I) C - Antigen-antibody complex disease (Type III) D - Antibody-mediated cytotoxic reactions (Type II) E - None of these

Antigen-Antibody Complex disease (Type III) In type III reactions, Ag-Ab complexes are deposited on the endothelium, stimulating complement and a neutrophilic inflammatory response and vascular damage. Look for localized multisystemic vasculitis. Classic Type III Disease include: -Glomerulonephritis -Hypersensitivity pneumonia (think moldy hay) -Purpura hemorrhagica (think post-strangles) -Anterior uveitis

A client with a small backyard flock presents a chicken carcass for necropsy. She says the bird had lost significant weight, was slow to move around, and was limping. Necropsy reveals various-sized granulomatous lesions distributed throughout the spleen, liver, intestines, and bone marrow, as seen below. What is the presumptive diagnosis? A - Infectious coryza B - Colibacillosis C - Candidiasis D - Ulcerative enteritis E - Avian tuberculosis

Avian tuberculosis This is classic avian tuberculosis (usually caused by Mycobacterium avium subsp. avium) which can affect all types of birds. It is a chronic, granulomatous bacterial infection that is slow to spread and is characterized by gradual weight loss. M. avium is prevalent worldwide and is spread by infective feces. It is zoonotic in immune-comp people. Dx: difficult. Positive wattle or skin tuberculin test confirms exposure but negative does not rule out disease. Suspicion can be based on seeing lots of acid-fast bacteria on smears from lesions. Rx: none. Prevent by careful purchasing and/or quarantine for at least 6 months of new birds. Improved sanitation and rapid flock turnover in commercial flocks has mostly eliminated this infection from these groups Candidiasis is an intestinal mycotic disease typically characterized by thickened mucosa and white, raised pseudomembranes in lesions in the crop and esophagus. Colibacillosis (E. coli) is common in chickens and can cause subacute pericarditis, airsacculitis, salpingitis, peritonitis, and acute fatal septicemia. It can occasionally cause granulomas in the liver and spleen (but not the intestine or bone marrow like in this chicken). Lameness would also be less common in the Hx for colibacillosis. Infectious coryza is a respiratory disease of chicken that acutely causes nasal discharge, sneezing and facial swelling Ulcerative enteritis (Clostridium colinum) causes depression, anorexia, and diarrhea.

A female veterinarian is pregnant and her physician tests her serologically for toxoplasmosis. IgM is negative. IgG is positive. What is the most appropriate interpretation? A - Mother at risk, baby is safe B - Both mother and baby at risk C - Mother is safe, baby at risk D - Both mother and baby are safe E - Need to re-check in 2 weeks for rising titers

Both mother and baby are safe Toxoplasmosis organism causes birth defects in a developing fetus if a mother is infected for the first time in her life while pregnant (IgM positive while pregnant - still requires a confirmatory test) Toxoplasmosis is not generally dangerous to immune-competent people and a positive IgG result suggests a previous infection.

Mitral regurgitation is the number one cause of congestive heart failure in dogs. Which breeds are predisposed to mitral regurgitation? A - Wirehaired fox terrier, Border collie B - Doberman pinscher, Boxer C - Great Dane, Afghan hound D - Miniature schnauzer, German shepherd E - Cavalier King Charles spaniel, Cocker spaniel

Cavalier King Charles Spaniel, Cocker Spaniel Think of mitral regurgitation/insufficiency in a Cavalier King Charles spaniel of any age, or in old male cockers with a hx of cough, labored breathing and exercise intolerance. Remember that COPD with fibrosis can cause similar signs - primarily older animals, and primarily westies. 4 other big cardio predispositions: - intermittently fainting boxer -> Boxer cardiomyopathy - rapid, irregular HR in a middle-aged, depressed, coughing, exercise-intolerant Doberman -> dilated cardiomyopathy (DCM) - Sick sinus syndrome (SSS) when you hear "fainting female min. Schnauzer" (also in dachshunds, cockers, west highland whites) - Tetralogy of Fallot in a young bulldog, Keeshond, wire haired fox terrier with cyanosis and exercise intolerance

A stray dog is presented after being hit by a car. The dog has hypotonic forelimbs and spastic paresis in the hinds. All four limbs have proprioceptive deficits and sensation loss-signs are worse in the forelimbs. Where is the lesion? A - Cervicothoracic: C6-T2 B - Thoracolumbar T3-L3 C - Lumbosacral L4-S3 D - Cannot say without cutaneous trunci reflex results E - Cranial cervical: C1-C5

Cervicothoracic: C6-T2 Weak, hypotonic (LMN) forelimbs and spastic paresis (UMN) hindlimbs are signs of a cervicothoracic lesion. May see worse signs in the fores than hinds. Note: This is the opposite presentation as Schaffer-Sherrington Syndrome (Severe acute spinal cord trauma in the region of T3-L3, where in lateral recumbency, the thoracic limbs are rigid and extended and the pelvic limbs appear flaccid in comparison). However, pelvic limb reflexes are normal to increased, as would be expected with an UMN lesion. Can localize T3-L3 by checking the cutaneous trunci reflex. C1-C5 would expect UMN signs in all four limbs, usually worse in the hinds. T3-L3 would see UMN in hind limbs and normal forelimbs.

A stray dog is presented after being hit by a car. The dog has hypotonic forelimbs and spastic paresis in the hinds. All four limbs have proprioceptive deficits and sensation loss-signs are worse in the forelimbs. Where is the lesion? A - Cervicothoracic: C6-T2 B - Cannot say without cutaneous trunci reflex results C - Lumbosacral L4-S3 D - Cranial cervical: C1-C5 E - Thoracolumbar T3-L3

Cervicothoracic: C6-T2 Weak, hypotonic (LMN) forelimbs and spastic paresis (UMN) hindlimbs are signs of cervicothoracic (C6-T2) lesion. May see worse signs in fores than hinds. Can localize T3-L3 lesions by checking the cutaneous trunci reflex. C1-C5 would expect UMN signs in all four limbs, usually worse in hinds. With a T3-L3, would see UMN hindlimb signs and normal forelimbs

Over the past two weeks, several pigs in a herd have been febrile and depressed. Many were constipated, then had diarrhea. A few were incoordinated and one had seizures. A few have died. Necropsy revealed petechial hemorrhages on the kidneys and larynx, and a hemorrhagic urinary bladder. Of the following choices, which one is most consistent with a presumptive diagnosis? A - Glasser's disease B - Streptococcus suis infection C - Swine dysentery D - Classical swine fever E - Erysipelas

Classical Swine Fever (CSF, "Hog Cholera"). Virology is required to confirm and differentiate from other causes of febrile hemorrhagic diseases. The best tissues to submit are tonsils, maxillary and submandibular lymph nodes, mesenteric lymph nodes, spleen, ileum and kidney. Glaesserella parasuis ("Glasser's Disease") is an acute bacterial infection, characterized by different combinations of meningoencephalitis, polyserositis and polyarthritis and can contribute to bacterial pneumonia.

A five-year-old female spayed domestic short-haired cat is presented with chronic vomiting, anorexia, and weight loss. Physical examination reveals icterus, dehydration, and fever. Thickened small intestinal loops are appreciated with abdominal palpation, and serum biochemistry shows increased bilirubin and liver enzymes. The cat is diagnosed with extrahepatic biliary obstruction with accompanying cholangitis and inflammatory bowel disease. Which one of the following normal anatomic features may predispose cats to this triad of diseases? A - Decreased diameter and increased branching of the hepatic ducts in comparison to other species B - Decreased proportion of alpha-smooth muscle actin in muscle fibers of the wall of the gall bladder in cats C - Enlarged duodenal papilla in comparison to other species D - Narrowing of the duodenum distal to the entry of the common bile and pancreatic ducts E - Common opening of the pancreatic and common bile ducts into duodenum

Common opening of the pancreatic and common bile ducts into duodenum In cats, there is a common opening of the pancreatic and common bile ducts into the duodenum. It is thought that this may predispose them to ascending cholangitis and pancreatitis after vomiting associated with IBD resulting in extrahepatic biliary obstruction. Other possible etiologies of EHBO include neoplasia, biliary stricture, duodenal obstruction, diaphragmatic hernia, and parasitic infection. In all cases, there is a lack of bile entry into the intestinal tract, leading to decreased absorption of fat and fat soluble vitamins such as Vit K, potentially resulting in coagulopathies. Except in transient cases that are related to acute pancreatitis, surgical intervention to relieve the obstruction is required as well as appropriate supportive care, antimicrobial treatment, and vitamin K administration.

A nine-year-old male neutered German shepherd is presented with unchecked bleeding from a cut on the gums above the right canine tooth. The owner relates that the dog has lost weight and had an episode of collapse three days ago, but he recovered. On physical exam, the gums are pale with petechiae and ecchymotic hemorrhages. There is tachycardia and a palpable cranial abdominal mass. A coagulation profile shows the following: Thrombocytes= 82,533 per microliter. [N=200,000-900,000]Buccal mucosal bleeding time (BMBT), increasedActivated partial thromboplastin time (aPTT), increasedProthrombin time (PT), increasedFibrin degradation products (FDPs), increased What disorder of coagulation best fits this pattern? A - Idiopathic thrombocytopenia B - Hepatic insufficiency C - Anticoagulant rodenticide toxicity D - Von Willebrand's disease E - Disseminated intravascular coagulation

Disseminated intravascular coagulation A pattern of low platelets, increased bleeding time and across the board increases in aPTT, PT, and FDP tests suggest DIC. DIC is not a disease in its own right. It's a complex hemostatic defect characterized by inappropriate activation of coagulation and deposition of fibrin within the microvasculature. The depletion of clotting factors and platelets due to this clot formation can result in widespread hemorrhage. If DIC is suspected it is essential to identify the underlying cause. Many disease can result in DIC, but the more common conditions include: sepsis, advanced heartworm disease, immune-mediated hemolytic anemia, anaphylaxis, envenomation, heat stroke, and severe fulminant pancreatitits. DIC is seen more commonly in dogs than in cats. Chronic DIC may be seen in dogs with hemangiosarcoma.

A two-day-old pup has a GGT level of 500 times the adult upper reference range. Which one of the following choices is the most likely interpretation? A - Related to bone development B - Due to colostrum absorption C - Caused by traumatic placental detachment D - Associated with neonatal cholestasis E - Suggestive of a congenital liver shunt

Due to colostrum absorption In newborn pups, lambs, and calves, an increase in GGT put to 1000x is normal where they receive high levels of colostrum from the dam. High levels of GGT are produced in the mammary epithelium during lactation.

An outbreak of diarrheal disease of piglets has occurred which affected the healthiest animals in the herd, one to two weeks after weaning. Some affected piglets had no signs except peracute death. Other affected piglets exhibit diarrhea, ataxia, paralysis, and recumbency. What condition is at the top of the differential diagnosis list? A - Porcine proliferative enteritis B - Hemagglutinating encephalomyelitis virus (HEV) C - Edema disease D - Epidemic transmissible gastroenteritis (TGE) E - Clostridium perfringens type C enteritis

Edema disease is a caused by shiga toxin producing E. Coli (STECT) Look for severe acute illness ranging from Peracute death with no signs to CNS involvement with ataxia, paralysis, and recumbency in healthiest pigs 1-2 weeks after weaning. HEV is almost exclusive to piglets less than 4 weeks old. Two clinical presentations: vomiting and wasting disease (VWD) and encephalitic. C. Perf type C enteritis (aka enterotoxemia in other animals) is characterized by hemorrhagic diarrhea in 1-3 day old piglets. PPE is principally a diarrheal disease of growing finishers (40-80 lb) and young breeding pigs. TGE in non-immune pig herds characterized by high morbidity and high mortality in piglets less than 1 week old.

A canine vaginal smear looks like the image below through a microscope. At what stage of the estrous cycle is this dog? A - Cannot tell from this slide B - Proestrus C - Anestrus D - Estrus E - Diestrus

Estrus Think "Corn flakes" with estrus. Look for greater than 90% cornified superficial cells (angular, sharp edges, tiny pyknotic nuclei or no nuclei) when you think canine estrus. And no neuts. In contrast, diestrus vaginal smeans will have more than 10% nucleated, round-edged parabasal and intermediate cells. Look for the reappearance of some neutrophils (variable numbers). Full cornification usually coincides with receptivity. Serum progesterone can predict the LH surge and help estimate ovulation. Rule of thumb: Breed bitch as soon as she will allow a male to mate or as soon as you see greater than 90% cornified superficial epithelial cells in a vaginal smear. Typically breed every 2-4d until bitch enters diestrus for maximum fertility success.

A five-year-old female spayed cat is presented with a two-day history of vomiting and anorexia. Physical exam reveals 5% dehydration and abdominal discomfort. Plain abdominal radiographs are shown below. Which choice is the most appropriate step to take next? A - Specific feline pancreatic-lipase assay B - Exploratory laparotomy C - Non-iodinated contrast series D - Gastroduodenoscopy with biopsy E - IV fluids and broad-spectrum antibiotics

Exploratory laparotomy is the most appropriate next step for this linear foreign body. On lat rads, there is an area of the SIs with very small gas bubbles where the walls of the SI appear undulating. In this case, the FB was a bikini string. A linear FB is a surgical emergency due to potential intestinal perforation and septic peritonitis. In severe cases, the "sawing action" of the linear FB against intestinal mucosa can cause extensive damage requiring extensive resection and anastomosis. Occasionally, the intestinal damage is too extensive and euthanasia is the best course. If a definitive Dx is not event on radiographs, consider abdominal US to further investigate the GIT.

A five-year-old female spayed Cocker Spaniel is presented with a drooping right ear and lip. There is ptosis O.D. (right eye) and the dog is drooling on the exam table. She is otherwise bright, alert, and responsive. What anatomic structure is damaged? A - Left side inner ear B - Left side medulla, motor tract C - Trigeminal nerve D - Facial nerve E - Right side inner ear

Facial Nerve Think of facial nerve paralysis (CN 7) with a unilaterally droopy face. Remember the facial nerve is motor to the muscles of facial expression (explaining the right side drooped ear, lip and eyelid) and innervated the lacrimal and salivary glands. Loss of innervation can lead to dry eye, and possible to exposure keratitis if animal loses the ability to close eyelid from damage to facial nerve innervation of the orbicularis oculi muscle. Idiopathic in 75% of canine cases and 25% of cats. You can also see these signs with middle ear damage (otitis media), from facial nerve trauma (ear surgery in dogs or pressure from halter buckles in anesthetized horses), or neoplasia. Think more of a dropped jaw with trigeminal nerve neuropathy (CN 5 - dogs, horses)

What behavior is the mare displaying in the image below? Image A - Windsucking B - Aggression C - Lignophagia D - Flehmen E - Cribbing

Flehmen response behavior seen in the following circumstances: normal stallion response to pheromones, normal response of all horses to new smells, sign of colic, or associated with granulosa cell tumors in mares. many other animals can also show this response

Which diseases are classically associated with a high anion gap (AG)? A - Diabetes mellitus, pancreatitis, proliferative enteritis B - Grain overload, ethylene glycol toxicity, renal failure C - Hypercalcemia of malignancy, hypoadrenocorticism, displaced abomasum D - Prostatic adenocarcinoma, pleural effusion, lymphangiectasia E - Eosinophilic enteritis, hypothyroidism, gastrinoma

Grain overload, ethylene glycol toxicity, renal failure Typically, an increased anion gap with: - Diabetic ketoacidosis - Renal failure - Ethylene glycol toxicity - Lactic acidosis from grain overload or strenuous exercise Decreased anion gap is uncommon. Think hemodilution, hypoalbuminemia, hypercalcemia.

A four-year-old Rat Terrier is presented with a three day history of progressive stumbling and falling. Physical exam reveals a right head tilt, left sided hypermetria, generalized ataxia and vertical nystagmus. Which one of the following choices is at the top of the differential list? A - Fibrocartilagenous embolism B - Central pontine myelinolysis C - Granulomatous meningoencephalitis D - Amyotrophic lateralizing sclerosis E - Ascending and descending myelomalacia

Granulomatous meningoencephalitis The lesion in this case localizes to the left cerebellum causing the left hypermetria and right paradoxical head tilt. Another top differential would be a cerebellar neoplasm.

Retained placenta and metritis can predispose to which one of the following secondary conditions in horses? A - Colic B - Cystic endometriosis C - Contagious equine metritis D - Postpartum dysgalactia syndrome E - Laminitis

In horses, retained placenta and septic metritis can lead to acute laminitis. There are four main types of laminitis: 1. Sepsis/SIRS-related (following septic metritis, pleuropneumonia, colitis, grain overload, or black walnut/CHO overload in research settings) 2. Endocrinopathic (horses with insulin dysregulation/equine metabolic syndrome or equine Cushing's disease; also associated with lush spring grasses). 3. Traumatic ("Road founder") 4. Support-limb (e.g. severe lameness on one limb requiring over-dependency on opposite leg, which then develops laminitis.

Which one of the following choices includes the cardinal sign of trigeminal neuritis? A - Circling and head tilt toward side of lesion, no other signs B - Masseter muscle pain associated with chewing C - Paralyzed eyelid, ear or lip on one or both sides of the face D - Inability to close the mouth E - Dysphagia, dysphonia and stridor

Inability to close the mouth Idiopathic trigeminal neuritis due to inflammation of CN5 is characterized by acute onset of flaccid jaw paralysis. Affected animals cannot close their mouth and have difficulty eating and drinking. Seen occasionally in dogs, rare in cats. Cause is unknown. Idiopathic facial nerve paralysis from CN7 damage results in the inability to move the eyelid, lip or ear and dryness of the eyes and mouth. Mastication myositis is characterize by pain on opening the mouth and swelling of the muscles of mastication (acute) or atrophy of the temporal is and mass enter muscles with the inability to open the mouth due to fibrosis (chronic) Circling and head tilt toward the side of the lesion with no other signs is a common presentation of vestibulocochlear nerve, cranial nerve 8 lesions. Concurrent CN7 paralysis and Horner's syndrome (ptosis, mitosis, enophthalmos) may be present with middle and inner ear infections.

What is the presumptive diagnosis in a chicken with the following necropsy finding? A - Avian spirochetosis B - Infectious bursal disease C - Histomoniasis D - Marek's disease E - Renal dystrophy

Infectious Bursal Disease (IBD Virus = IBDV) Note the enlarged, balloon-like hemorrhagic bursa of Fabricius. This is caused by Birnavirus, shed in feces and transferred barn to barn via forties. Very stable in the environment and difficult to eradicate from premises. Signs include an enlarged, balloon-like hemorrhagic bursa in older birds or the MORE IMPORTANT subliminal form in young birds which causes immunosuppression via destruction of immature lymphocytes in the bursa, thymus and spleen. Immunosuppressive birds don't respond well to vaccination and are predisposed to infections with normally non-pathogenic viruses/bacteria. Common disease usually exacerbated by IBDV infections.

A seven-week-old pot-bellied pig is presented with a posterior swelling that looks like this: Which one of the following is the most likely clinical diagnosis? A - Testicular torsion B - Intersex syndrome C - Perineal hernia D - Inguinal hernia E - Cryptorchidism

Inguinal Hernia A common problem in pigs, Tx surgically: midline skin incision, cranial to scrotum; ligate and excise vas deferens, blood vessels. Both inguinal ring areas should be closed to prevent herniation post-Sx. Removal of tunic, cremaster muscle, extra SQ tissue, with closure to obliterate empty space helps prevent seromas. Intersex syndrome is described in pigs and goat (rare) but look for BOTH genitalia. Perineal hernia more a problem of middle-aged pure bred dogs. Cryptorchidism a testicle is retained, not protruding. Inguinal hernia in horse: https://www.merckvetmanual.com/digestive-system/colic-in-horses/overview-of-colic-in-horses

What is the neuroanatomic cause of extensor rigidity in the forelimbs of a patient with Schiff-Sherrington posture? A - Subdural white matter compression at C6-T3 B - Interruption of inhibitory neuron input from lumbar spinal cord C - Brachial plexus trauma D - Polyradiculoneuritis of peripheral nerve sheaths at cervical intumescence E - Damage to thoracic spinocerebellar tracts in superficial white matter

Interruption of inhibitory neuron input from lumbar spinal cord Schiff-Sherrington posture occurs due to interruption of inhibitory neuron input from the lumbar spinal cord. The posture is characterized by rigid forelimb extension with normal function, with concurrent hindlimb paresis and paralysis. Trauma to the spinal cord between T3-L3 results in inhibitory pathway interruption. Neurons caudal to the cervical intumenscence are "released", causing excessive extensor tone in the forelimbs. This posture is not associated with prognosis of the underlying disease. Think of Schiff-Sherrington posture when you see a combination of forelimb extensor rigidity and hindlimb flaccid paralysis in an animal with spinal trauma, like being hit by a car or acute intervertebral disc disease.

What does this finding mean in a cat? A - Lingual polyp, can remove, but recurrence is common B - Cuterebra infestation C - Female in estrus D - It is an intact male E - Nasopharyngeal polyp, cat will be fine after surgical removal

It is an intact male The penis of an intact male demonstrates backward-projecting cornfield spines on the glans due to the presence of circulating androgens. Penile spine scare absent in neutered male cats. The presence of spines can help to diagnose cryptorchid cats. Spines should disappear within 6 weeks of castration. Penile spines help toms fertilize queens by breaking through copulatory plugs. This is helpful in a species in which sperm competition inside the female can be fierce.

What recommendation should be given to the owner of a mare that has had Caslick's vulvoplasty surgery? A - Mare will need an episiotomy before foaling B - Mare should not be bred by a stallion C - Mare has a higher than normal risk for metritis D - Mare has a decreased risk for perineal laceration E - Higher risk of uterine torsion

Mare will need an episiotomy before foaling (Surgically planned incision on the perineum and the posterior vaginal wall) is needed prior to foaling and may be needed prior to breeding by stallion if the remaining valvular cleft is too small to permit intromission. A Caslick operation (vulvoplasty, a superficial form o the episioplasty) is used to treat pneumovagina in horses to prevent genital infections and infertility.

A high-producing dairy cow that freshened three weeks ago is off-feed. On physical exam her heart and respiratory rates are within normal limits. T=101.2F (38.4 C) [N=100.4-102.8 F; 38.0-39.3 C] Rumen motility is decreased and a urine test for ketone bodies is positive. There is no evidence of mastitis and the uterus is clear of infection. On the left side, a high-pitched musical "ping" is audible via stethoscope during percussion over the ribs on a line between the elbow and tuber coxae (hip). What acid-base abnormality is most likely in this cow? A - Respiratory acidosis B - Metabolic acidosis C - Respiratory alkalosis D - Metabolic alkalosis E - Depends on severity of displacement

Metabolic alkalosis Think of hypochloremic metabolic alkalosis due to hydrochloric acid sequestration in the abomasum of a cow with a displaced abomasum. Abomasal hypomotility, ongoing HCl secretion into the abomasum, and partial abomasal outflow obstruction all contribute. Metabolic alkalosis can also be due to HCl loss in monogastric animals who vomit. In a similar way, metabolic acidosis can be due to HCO3 loss in saliva if the animal can't swallow, or from diarrhea.

A one-year-old female llama is presented with hyperkeratosis around the nose and mouth. What is the most likely diagnosis? A - Wry face B - Dorsal nasal alopecia C - Dermatophilosis D - Munge E - Zinc-responsive dermatosis

Munge (idiopathic nasal/perioral hyperkeratotic dermatosis) Some affected animals have lesions on the bridge of the nose and around the eyes and ears. Average age of onset is 6 mo.- 2 years old. Rx: address secondary bacterial infections (e.g. daily 10% povidone iodine scrubs and apply 7% tincture of iodine) If minimal response, include a topical glucocorticoid or intralesional triamcinolone acetonide. If still no response, evaluate immune function. Dorsal nasal alopecia is characterized by alopecia over the bridge of the nose with normal to variably scaly, hyperpigmented, and thickened skin. It's more common in dark haired animals and can be secondary to fly bites or rubbing the nose. Animals with idiopathic hyperkeratosis (Zn-responsive dermatosis) present with thickened, nonpruritic papules with tightly adherent crusts in the less densely haired areas of the perineum, ventral abdomen, inguinal region, medial thighs and Axilla.

A three-year-old white rat is presented with yellowing fur and a brownish, granular sebaceous secretion at the base of affected hair shafts. What should the owner be told? A - Normal age-related change B - Common manifestation of chorioptic mange C - Moisture-related dermatitis, cage needs more frequent cleaning D - Staph dermatitis, responsive to antiseptic shampoo E - Probable ringworm, resolves spontaneously with sunlight exposure

Normal age-related change Rats normally live 2-3 years, though some may live to 5 yrs off age. With age, the hair coat of white rats normally develop yellowing fur. They may also show a brownish, granular sebaceous secretion at the base of affected hair shafts which an owner may mistake for a skin parasite.

Which group is most at risk of developing the lesion evident in this image? A - Cattle without reticular magnets; Charolais B - Young horses ingesting Perilla mint; Standardbreds C - Sheep or goats pastured above 2000 meters; Saanens D - Old, small-breed dogs; Cavalier King Charles Spaniel E - Obese cats, any breed; Burmese cats

Old, small breed dogs; cavalier King Charles spaniel This is endocardiosis (degenerative valve disease). Note the modular thickening of the mitral valve margins. Degenerative valve disease is the most common cardiac disease of dogs, accounting for about 75% of all canine cardiovascular disease. Roughly 60% of affected dogs have myxomatous degenerative of the mitral valve. Older, small-breed dogs have a higher incidence and cavalier King Charles spaniels are prone. Don't confuse with infective endocarditis, which is due to bacterial infection of the cardiac valves and endocardia muscle.

An eight-year-old female Saint Bernard is presented with right pelvic limb lameness and swelling above the hock. Which one of the following choices is the most likely form of neoplasia based on radiographic appearance and incidence of occurrence? A - Melanoma B - Hemangiosarcoma C - Lymphosarcoma D - Osteosarcoma E - Fibrosarcoma

Osteosarcoma This is a good reminder that osteosarcoma can occur almost anywhere, in spite of the mnemonic that osteosarcoma usually occurs away from the elbow and near the knee. There is a large, expansile destructive osseous lesion of the distal right tibia with cortical destruction most severe at the caudal and medial margins. .There is also mild, irregular periosteal proliferation and soft tissue thickening at the site. This lesion is characteristic of a primary bone tumor. The location and expansile destructive pattern are features to look for. A fungal osteomyelitis would tend to have more productive bone and is often multifocal.

A three-year-old Irish setter is presented with a history of lameness and swelling in the distal forelimb. A radiograph is shown below. What is the presumptive diagnosis? A - Panosteititis B - Osteosarcoma C - Hypertrophic osteodystrophy D - Osteomyelitis E - Osteochondrosis

Osteosarcoma, a common, aggressive bone tumor typically found in the appendicular skeleton, especially distal radius. Common locations of osteosarcoma: away from the elbow, near the knee. 90% have microscopic metastases to lungs by the time of Dx (less than 10% will have visible thoracic metastasis at time of diagnosis). Usually, they do not cross the joint (unlike osteomyelitis which usually DOES cross the joint). Look for soft tissue swelling, periosteal proliferation, sunburst periosteal reaction (33%), possible pathologic fractures.

A practice is using an FeLV test with a sensitivity of 90% and a specificity of 95%. Assuming the prevalence of feline leukemia in the area is 5%, what is the predictive value negative (PVN) of the test? A - 75% B - 99% C - 88% D - 45% E - 55%

PVN is 99% The trick with this kind of question is to pick an imaginary number of animals that you test (e.g. 1000) If prevalence is 5%, then 50/1000 cats have FeLV and 950 cats are disease-free. A 90% sensitivity means that 45/50 will be false negatives. A 95% specificity means 902/950 are false positives. PVN = true positives / (false positives + true positives) = 902/ (902+5) = 99%

A two-year-old male neutered domestic shorthair cat is presented after a suspected fall from a height of approximately 35 feet (over ten meters). The cat is initially stabilized with intravenous fluids and oxygen therapy. Four hours later, the cat has not urinated and no urinary bladder is palpable. An abdominal focused assessment with sonography for trauma, triage, and tracking (AFAST) reveals a moderate volume abdominal effusion, with no visible urinary bladder. What test would confirm the top differential? A - Paired plasma and effusion potassium and creatinine B - Abdominocentesis with Gram stain and cell count C - Lactate and glucose levels of peritoneal sample and blood D - Total protein and microscopic analysis of the ascites E - Measurement of BUN in blood and fluid

Paired plasma and effusion potassium and creating. This cat has a traumatic urinary bladder rupture (Uro abdomen/-peritoneum) best diagnosed with paired serum and effusion potassium and creating measurements. Uroperitoneum is a surgical emergency. In cats, a fluid to blood creatinine ratio of 2:1 (same in dogs, foals) and fluid to blood potassium ratio of 1.9:1 (1.4:1 in dogs, not done in foals) is diagnostic for Uro abdomen. BUN measurements is often not helpful because it readily equilibrates across the peritoneum. "High rise syndrome" is a common poly trauma in cats, characterized by a triad of thoracic, cranial facial, and limb injuries. Stabilize life threatening thoracic trauma such as pneumothorax, pulmonary contusions, and diaphragmatic hernia before surgical treatment of orthopedic trauma. Uroperitoneum also occurs in newborn foals (more common in males, often associated with birth or prolonged recumbency in the NICU). Affected foals typically present within a couple of days of birth with abdominal straining and distention, tachypnea, and decreased nursing with depression. Labs: hyponatremia, hypochloremia, hyperkalemia, azotemia.

Failure to control mild endemic respiratory disease in swine caused by Mycoplasma hyopneumoniae predisposes pigs to complications. Which other organism works in conjunction with Mycoplasma hyopneumonia to cause exudative bronchopneumonia and polyarthritis? A - Fusobacterium necrophorum B - Pasteurella multocida C - Bordetella bronchiseptica D - Swine influenza virus E - Hemophilus parasuis

Pasteurella multocida infection in conjunction with Mycoplasma hyopneumoniae causes exudative bronchopneumonia, polyarthritis, and chronic lung lesions. Mycoplasma hyopneumoniae (also called "enzootic pneumonia") is a common, smoldering, low-level illness. Stressors (parasites, other infections, even weather) can result in severe pneumonia. Best managed by decreasing stressors with improved ventilation and decreased overcrowding. In endemic herds, ABX for sick individuals (ie: lincomycin, tylosine, tiamulin, or a tetracycline) helps control illness, most likely by preventing secondary bacterial infection (like pasteurella). Bacterin vaccines gives good protection, decreases signs (coughing), pre-farrowing vaccination of sows decreases colonization in suckling piglets.

An adult horse with clinical signs of voluminous gastric reflux, depression, colic, and fever is most likely suffering from which condition? A - Proliferative enteropathy B - Right dorsal colitis C - Proximal enteritis D - Cantharidin toxicity E - Ulcerative duodenitis

Proximal enteritis Proximal enteritis (duodenitis-proximal jejunitis) is a clinical syndrome characterized by large volumes of gastric reflux resulting from excessive fluid and electrolyte secretion into the small intestine and small intestinal inflammation and edema. Laminitis is an important potential sequela. The cause is unknown but several bacteria and toxins including Clostridioides difficile (formerly Costridium difficile), C. perfringens, Salmonella, and fumonosin B1 mycotoxins have been implicated. Ulcerative duodenitis is a disorder of foals resulting in fever, colic, diarrhea, and delayed gastric emptying. Lawsonia intracellularis causes proliferation enteropathy in foals and weaklings, a disease characterized by hypoproteinemia, diarrhea, chronic ill thrift, and ventral edema. Right dorsal colitis is typically a result of non-steroidal anti-inflammatory drug toxicity and results in hypoproteinemia and colic. Cantharidin toxicity (blister beetles, Epicauta spp.) causes a wide range of clinical signs predominated by profuse diarrhea, stranguria and pollakiuria, and colic.

Under what conditions is a very sensitive test used? A - Lethal disease; highly prevalent disease B - Common disease; infectious diseases C - Treatment does not affect prognosis; non-infectious diseases D - Zoonoses; untreatable diseases E - Rare disease; early diagnosis improves prognosis

Rare disease; early diagnosis improves prognosis. You need a very sensitive test if: 1. Disease is rare (e.g. BSE) 2. Early Dx improves prognosis (e.g. HIV in people) 3. The disease is highly lethal or consequences of missing a case are severe (e..g rabies, brucellosis, BSE, screw-worm, FMD, EIA) Remember that a highly sensitive test will have very few false negatives. That means if a test is highly sensitive, you can trust a negative test.

What condition is associated with rectal stricture in pigs? Salmonella typhimurium - Salmonella typhimurium Rotavirus - Rotavirus Coccidiosis - Coccidiosis Intussusception - Intussusception Intestinal spirochetosis - Intestinal spirochetosis

Salmonella typhimurium is associated with rectal strictures in growing pigs. Caused by an ulcerative proctitis that damages rectal tissue. Can see large numbers of cases. Can see sporadic rectal strictures as a sequelae to rectal prolapse. Follow this link to see a Merck image of rectal prolapse in a sheep. Intestinal spirochetosis is a post-weaning diarrhea seen in the ABSENCE of Brachyspira (Serpulina) hyodysenteriae (swine dysentery), but similar in presentation to it. This syndrome is being recognized more frequently worldwide.

What condition is associated with rectal stricture in pigs? A - Intussusception B - Coccidiosis C - Rotavirus D - Intestinal spirochetosis E - Salmonella typhimurium

Salmonella typhimurium is associated with rectal strictures in growing pigs. Caused by ulcerative proctitis that damages rectal tissue. Can see sporadic rectal strictures as a sequelae to rectal prolapse. Intestinal spirochetosis is a post-weaning diarrhea seen in the absence of Brachyspira hyodysenteriae (swine dysentery) but similar in presentation to it.

During the necropsy of an eight-year-old mixed breed dog from the Southern United States, reactive granulomas in the esophagus containing bright red worms, 40 mm to 70 mm long are found. Which choice is the most likely diagnosis? A - Gastrophilus spp. B - Haemonchus placei C - Spirocerca lupi D - Ollulanus tricuspis E - Physaloptera spp.

Spirocerca lupi A disease of dogs in the Southern U.S. and tropical climates, Spirocerca lupi (esophageal worms) make reactive granulomas of variable size in the esophageal, gastric or aortic walls. Spirocercosis may also lead to aneurysm in the thoracic aorta or an ossifying spondylitis of the posterior thoracic vertebrae. Typically asymptomatic, but large granulomas can cause esophageal obstructino. Large granulomas may become neoplastic (osteosarcoma, fibrosarcoma) Some dogs develop spondylitis or enlargement of the extremities characteristic of hypertrophic osteopathy. All of the other choices are gastric parasites. Ollulanus tricuspis is an uncommon gastric parasite of cats. Physaloptera spp. is a stomach worm that may cause vomiting, anorexia, dark feces in dogs and cats. Haemonchus, Ostertagia, and Trichostrongylus are found in the abomasum of ruminants. Gasterophilus spp. are the larvae of horse bot flies, found in the stomach of horses.

An adult male cat adopted from a shelter with an unknown vaccination history is presented. The cat tests positive for feline leukemia virus (FeLV) infection by IFA. What should the owner be told? A - Cat may be vaccinated for FeLV B - Possible transient FeLV infection C - Strong chance of lifelong FeLV infection D - Need an ELISA test in 12 weeks to confirm diagnosis E - Need a Western blot test to confirm diagnosis

Strong chance of lifelong FeLV infection 97% of cats positive for feline leukemia virus (FeLV) by immunofluorescent antibody assay (IFA) remain persistently infected and viremic for life. The FeLV ELISA is more Sensitive than IFA (fewer false Negs, so trust a Neg test more) so it is better for general screening test with which to start. The FeLV IFA is more specific than the ELISA (fewer false Pos, so trust a pos test more) so it is a better confirmatory test for any cats with a positive FeLV ELISA. Three Things to remember for FeLV vaccinations: 1. FeLV ELISA and IFA tests measure ANTIGEN, not antibody, so FeLV vaccination does NOT interfere with testing. 2. Vaccinate FeLV-positive cats yearly against respiratory, enteric viruses with INACTIVATED vaccines. 3. FeLV vaccination may be associated with sarcoma development. Vaccinate LOW on the LEFT hind limb.

A ten-year-old female spayed Persian cat is presented for evaluation of a bleeding lump. There is a two-cm diameter, firm, solitary, hairless, dome-shaped, ulcerated mass on her left neck just behind the jaw that is bleeding. Cytology obtained by fine needle aspirate reveals reveals cohesive clusters of large cells with round to oval nuclei that are tightly adherent to each other (see below). Based on these findings, which choice is the most appropriate treatment option? A - Wide excision of mass, remove regional lymph node, prednisolone, radiotherapy B - Cryosurgery, debridement, repeat in 3-5 weeks C - Immunotherapy, wide margin excision, chemotherapy D - Debulk, intralesional implant chemotherapy, minimize sun and cigarette smoke exposure E - Surgical removal

Surgical Removal This is a basal cell tumor. Basal cells make up the base of the epidermis, and a benign growth of these is called a basal cell tumor or adenoma. Although this tumor is benign, surgical removal is the treatment of choice due to ulceration and secondary inflammation. Basal cell carcinomas (the malignant form) do not form raised masses but appear as ulcers on the head, neck, and legs: surgical removal is also recommended. Basal cell tumors are common in older cats, accounting for 15-26% of all feline skin tumors. Most are slow growing and typically present for months prior to diagnosis. DLH, Himalayans, and Persians are predisposed. Note the characteristic cytology findings of the epithelial neoplasia present in this case: cohesive clusters of large, round to polygonal cells, demonstrating distinctive cell borders that are tightly adherent to each other with round to oval nuclei. https://eclinpath.com/cytology/cytology-interpretation/

Fractures of the proximal sesamoid bones in horses are often associated with damage to which structure? A - Deep digital flexor tendon B - Superficial digital flexor tendon C - Superior check ligament D - Impar ligament E - Suspensory ligament

Suspensory Ligament The suspensory ligament is most likely to be damaged with fracture of the proximal sesamoids due to its insertion onto these bones. Proximal sesamoid fractures are relatively common, caused by overextension. The prognosis for return to soundness is often predicated on the extent of the damage to the suspensory apparatus. Proximal sesamoid fractures and their attendant suspensory apparatus problems are the number one cause of racetrack deaths.

A 22-year-old Thoroughbred mare is presented for bilateral green nasal discharge, retching, coughing, and inappetence noted after feeding this evening. The owner recently switched the mare to a pelleted diet because of the mare's poor dentition. Physical exam reveals bilateral green nasal discharge. The mucous membranes are pink and moist with a capillary refill time of 1.5 seconds. A slight swelling is notable in the proximal-left jugular groove just caudal to the vertical ramus of the mandible. manure Based on the most likely diagnosis, what initial therapy would be recommended? Value Normal 100.6°F (38.1°C) 99.1-100.8°F (37.3-38.2°C) HR=40 bpm 28-40 bpm R=12 brpm 10-14 brpm A - Withhold food and water; give sedatives and oxytocin; recheck in 2 hours B - Ultrasound thorax, perform transtracheal wash for cytology and culture, begin broad-spectrum antimicrobials C - Cautious guttural pouch endoscopy with lavage for Streptococcus equi spp. equi PCR, quarantine horse until results are available  D - Pass nasogastric tube; give atropine; lavage the esophagus until the obstruction clears E - Administer antimicrobials, refer horse immediately for endoscopy and lavage under general anesthesia

The most appropriate initial therapy for this systemically stable horse with an acute esophageal obstruction (also incorrectly called "choke") is to withhold food and water, give sedatives and oxytocin (and n-butylscopolammonium bromide) to relax the esophagus, and recheck in a couple hours. This is often successful in uncomplicated cases. If obstruction has not resolved within 2-4 hours, sedate to lower the head, pass a nasogastric tube to the level of the obstruction, and GENTLY lavage the esophagus with water. This is typically successful. In cases that do not resolve with thisTx, refer for ultrasound, esophageal endoscopy, and/or general anesthesia with esophageal lavage at a referral center. Click here to see an endoscopic image of an esophagus after resolution of a 12-hour obstruction. Obstructions lasting more than several hours require IV fluids and electrolytes, anti-inflammatories, and antimicrobials (for secondary aspiration pneumonia). Complications include aspiration pneumonia, repeat obstruction, and esophageal stricture.

A four-year old dog is presented with a history of sudden onset of collapse and respiratory distress. Physical exam shows pale mucous membranes, jugular distention and a jugular pulse. On an echocardiogram, a mass of worms is noted in the right atrium and caudal vena cava. What should the owner be told? A - Need to hospitalize, treat medically (Melarsomine 3 dose protocol) B - This is vena cava syndrome, surgery is best option C - Prognosis is fair, must stabilize first, then reassess D - Need to hospitalize, treat medically (Ivermectin/heparin protocol) E - The prognosis is hopeless and euthanasia is strongly encouraged

This is vena cava syndrome, surgery is best option This dog is presented in severe acute heart failure with Class IV heart worm disease (aka caval or vena cava syndrome) The treatment of choice is surgical-worm removal from R heart and pulmonary artery via jugular vein using fluoroscopy and long flexible alligator forceps. Highly effective at reducing worm burden when performed by an experienced surgeon. Prognosis is still guarded and treatment is expensive. Most heart worm cases are treated with the "3-dose" protocol of Melarsomine. That's one dose now, then two doses 24 hours apart one month later.

A herd of ewes has had an abortion storm in which 80% of animals are affected. All stages of gestation are affected, and lambs that are born are very weak. On the placentas, cotyledons have white foci of necrosis. Which one of the following choices is the primary differential diagnosis? A - Listeria monocytogenes B - Chlamydiosis C - Histophilus ovis D - Brucella E - Toxoplasma gondii

Toxoplasma gondii T. gondii infection can cause severe abortion storms in sheep. Clinical signs pathognomonic of T. gondii are bright to dark red cotyledons with white foci of necrosis. The intercotyledonary areas of the placenta are normal. With Brucella and Histophilus ovis, the abortion rate is much lower. Abortion caused by Listeria spp. is associated with the feeding of silage.

A group of yearling stocker cattle on summer pasture are presented for coughing in approximately 30% of the group. The owner has treated the coughing animals with antibiotics with no response. When the herd is gathered for examination the coughs worsen and some animals become dyspnic. Some of the cattle look thin and have rough haircoats. Necropsy of one animal shows the following. What is the appropriate action for this herd? A - Treat the coughing animals with an anthelmintic B - Do nothing as this is a self-limiting problem C - Call the state vets office to report a possible foreign animal disease D - Check for moldy sweet potatoes and Perilla mint E - Treat the entire group with an anthelmintic

Treat the entire group with an anthelmintic Those not showing clinical signs may still be subclinically infected and benefit from treatment. Products used to treat lungworms also impact GI nematodes, so treating an entire group will impact GI nematode refugia and anthelmintic resistance. This should be factored into your treatment decisions. Lungworm infection (verminous pneumonia) caused by Dictyocaulus viviparous. It is seen mostly in yearlings on pasture in the summer and fall. The primary clinical sign is coughing. Dx is by finding larvae (not eggs) in feces or adult worms in the airways on necropsy. 4-Ipomeanol toxicity from moldy sweet potatoes or Perilla mint can present similarly but necropsy shows pulmonary edema and emphysema.

A dog hit by a car in the last three hours is presented recumbent and traumatized. Plain radiographs of the awake dog are shown below. Positioning is not perfect because the dog is in pain, but if there is a problem evident in the radiograph, in which quadrant does it lie? A - Looks normal B - Upper left C - Lower left D - Lower right E - Upper right

Upper Left You are looking at an iliac fraction in the upper left quadrant. Generally, when you assess pelvic rads after trauma, you are looking for pelvic fractures, femoral fractures, (especially head-and-neck fractures) or hip dislocation/luxation (usually cranio-dorsal). Sometimes, the priorities of triage require you to accept less-than-optimal radiograph positioning in the serve of rapid assessment. The visible space evident at the sacroiliac junction on the upper right is not a subluxation, it is simply poor positioning when taking radiographs on an awake dog.

A seven-year-old female chinchilla is presented with a history of anorexia and lethargy. Which one of the following organs has the most prominent pathology in these radiographs? A - Urinary bladder B - Stomach C - Normal radiographs for a chinchilla D - Uterus E - Colon

Uterus This chinchilla has pyometra. The abdomen is markedly distended with poor detail caused by organomegaly. There are two soft tissue tubular masses in the caudal abdomen along the lateral margins. The small bowel loops are displaced cranially and medially. Within the thorax, there are no abnormalities seen.

Several younger sheep in a large herd have died suddenly with a diagnosis of infectious necrotic hepatitis secondary to fluke infestation. Which choice is the most practical and effective control measure you can take next to prevent future cases? A - Clorsulon antihelmintic for whole herd B - Clostridium hemolyticum bacterin for animals under 3 years C - Treat ponds with copper sulfate molluscicide against lymnaeid snails D - High-dose penicillin for clinical cases E - Vaccinate all animals with Clostridium novyi toxoid

Vaccinate all animals with C. novyi toxoid. Active immunization with C. novyi toxoid before the late summer is the most effectvie way to control and prevent infectious necrotic hepatitis (Black Disease) in sheep. Because vaccinated sheep have long-term immunity after only one shot, only new introdutions to the flock (lambs and sheep brought in from other areas) need to be vaccinated. Reducing the number of snails (intermediate hosts for fluke cercaria) with molluscicides or by fencing off wet areas may not be practical due to expense or amount of pasture lost. Likewise, use of flukicides is complicated by the need for careful timing and long withdrawal times for meat and milk.

Which one of the following choices can be an underlying cause for the problem seen in this turtle? A - Nutritional secondary hyperparathyroidism B - Pasteurella multocida C - Mycoplasmosis D - Septicemic cutaneous ulcerative disease (SCUD) E - Vitamin A deficiency

Vitamin A Deficiency This is an aural abscess, which can occur secondarily to Vitamin A deficiency. Captive terrestrial box turtles are most at risk, usually due to diets containing little vitamin A. Other presentations of hypovitaminosis A include froth from the nose (a sign of respiratory disease) and renal disease.

One description of a typical heart sound is "lub-dub". What makes the first heart sound (S1) (i.e., the "lub")? A - Mitral and semilunar valves B - Aortic and pulmonic valves C - Atrioventricular valves D - Ventricular filling E - Atrial contraction

Atrioventricular valves The first heart sound (S1 - the "lub" in "lub-dub") is caused by the closure of the atrioventricular valves. The second sound (S2) is the closure of the aortic and pulmonic valves (semilunar valves) A third sound (S3) is the end of rapid ventricular filling and a fourth sound (S4) is atrial systole (atrial contraction). You can often hear all 4 sounds in horses, but typically hear only S1 and S2 in cattle and small animals.

An eight-month-old male intact German shepherd dog is presented for poor weight gain, chronic diarrhea, and polyphagia. Fasting serum trypsin-like immunoreactivity (TLI) measurement is <2.0 µgL (Normal = 5.7-45.2 µg/L). What is the most likely cause of this dog's clinical signs? A - Pituitary dwarfism B - Acute pancreatitis C - Inflammatory bowel disease D - Small intestinal bacterial overgrowth E - Acinar cell atrophy

Acinar cell atrophy This dog has exocrine pancreatic insufficiency (EPI) due to pancreatic acinar atrophy (PAA). PAA is the most common cause of EPI in German Shepherd dogs and is considered hereditary. It is also seen in rough-coated Collies and Eurasians. In cats and older dogs, EPI is most commonly caused by chronic pancreatitis. Serum TLI is the best test for EPI in dogs and cats. A significantly low (< 2.5ug/L) fasting TLI is diagnostic for EPI. In EPI, there is impaired synthesis and release of digestive enzymes by pancreatic acinar cells, resulting in maldigestion, weight loss/failure to gain weight, diarrhea, and polyphagia. Concurrent SI dysbiosis and cobalamin deficiency are common. Treat with oral enzyme replacement and cobalamin supplementation if deficient. Patients with EPI require life-long enzyme replacement. Prognosis is good with appropriate treatment.

Which canine parasite can cause cutaneous larva migrans in people? A - Habronema spp. B - Ancylostoma spp. C - Trichuris spp. D - Trichostrongylus spp. E - Spirocerca sp.

Ancylostoma (hookworms) May cause cutaneous larva migrants Roundworms (Toxocara, Toxascaris, bayliscaris) are also zoonotic, causing visceral and ocular larva migrants in people. In dogs, whipworms (Trichuris) are associated with hypoadrenocorticism-like syndrome (hyponatremia, hyperkalemia, azotemia, metabolic acidosis) Whip worm infections has been suggested as one cause of cecocolic intussusception. Habronema in horses can cause tumor-like stomach nodules and sometimes cutaneous lesions. Trichostrongylus can cause parasitic gastritis and enteritis in sheep, goats, and cattle.

Half of the flock from a local broiler poultry farm has died in the last ten days. Examination shows chickens that are gasping and coughing, extending their necks to inhale. Some have blood-stained beaks. There are no obvious neurologic signs. Necropsy finds hemorrhage in the airways, as shown below. What is the most likely presumptive diagnosis? A - Infectious coryza B - Marek's disease C - Viscerotropic velogenic Newcastle disease D - Infectious laryngotracheitis E - Fowl pox, diphtheritic form

Blood in the trachea and this history suggest infectious laryngotracheitis (ILT). A highly contagious herpesvirus infection, ILT is characterized by RESPIRATORY signs (rales, severe dyspnea, coughing). In severe forms, mortality can reach 50%, typically due to occlusion of the trachea by blood, mucus or caseous exudates. In most states ILT is REPORTABLE. Infectious coryza is a milder respiratory disease, with swelling around the eyes and head, sneezing, nasal discharge. Newcastle disease is characterized primarily by respiratory signs. Severe forms include depression, neurologic signs or diarrhea. Look for GI hemorrhage with most severe form, viscerotropic velogenic Newcastle disease (VVND), which is REPORTABLE. With fowlpox, only a few birds typically affected with scablike lesions around head. Click here to see fowlpox. See only depression before death with Marek's disease. Some birds may have characteristic unilateral paresis (one leg forward, one leg back). On necropsy, see lymphoid tumors in organs and enlarged nerves.

A seven-year old male German shepherd is presented with a history of weakness in the hind limbs, urinary incontinence and recent obsessive chewing around his tail area. Dorsiflexion of the tail over the back and extension of the hind limbs elicits a painful response. He does not withdraw each hind leg when a toe is pinched, but bears weight on the hindlimbs. Patellar reflexes are normal. What is the clinical diagnosis? A - Hip dysplasia B - Cauda equina syndrome C - Radiculoneuritis D - Diskospondylitis E - Wobbler syndrome

Cauda equina syndrome This is a common presentation of cauda equina syndrome. Look for PAIN in the lumbosacral area (elicited by tail raise, hindlimb extension), LMN hindlimbs, especially sciatic nerve damage at L7-S1 (lack withdrawal reflex) +/- urinary/fecal incontinence, +/- SELF-MUTILATION of tail, perineum, pelvic limb. Lesion due to compression of cauda equina at L7-S1 (lumbosacral stenosis). Can be congenital (abnormal development of dorsal arch L7-S1, small dogs, Border collies) or acquired (degenerative changes, big dogs, especially German Shepherd, Rottweiler, Boxer). Wobbler Syndrome (a.k.a. Cervical vertebral instability, (caudal) cervical spondylomyelopathy) is a congenital cervical spinal cord disease. Affected dogs are born with vertebral canal stenosis which can worsen later in life due to intervertebral disc disease, vertebral malformation and/or ligamentous hypertrophy, usually seen at C5-C6 or C6-C7 in Dobies > five years, Great Danes < two years. Diskospondylitis is a good second choice on your DDX of a large middle aged dog presenting with lumbosacral pain. Due to bacterial/fungal infection of intervertebral disk and adjacent vertebral bodies. Look for systemic signs like fever (1/3 patients), weight loss, anorexia.

Mitral regurgitation is the number one cause of congestive heart failure in dogs. Which breeds are predisposed to mitral regurgitation? A - Great Dane, Afghan hound B - Wirehaired fox terrier, Border collie C - Miniature schnauzer, German shepherd D - Doberman pinscher, Boxer E - Cavalier King Charles spaniel, Cocker spaniel

Cavalier King Charles spaniel, cocker spaniel Mitral regurgitation/insufficiency in cavalier King Charles spaniel of any age or in old male cockers with a Hx of cough, labored breathing and exercise intolerance. COPD with fibrosis can cause similar signs primarily in older animals and westies. 4 big cardio predispositions: - Intermittently fainting Boxer -> boxer cardiomyopathy - Rapid, irregular HR in middle-aged, depressed, coughing, exercise-intolerant Doberman -> dilated cardiomyopathy (DCM) - Fainting female min. Schnauzer -> Sick sinus syndrome (SSS) - Young bulldog, Keeshond, wire haired fox terrier with cyanosis and exercise intolerance -> Tetralogy of Fallot

A four-month-old mixed breed puppy is presented with a one-month history of generalized wobbliness when walking and head bobbing when attempting to eat.Owners do not think there has been progression of the signs. Based on the MRI findings, which one of the following choices is the most likely diagnosis? A - Lead toxicosis B - Cerebellar hypoplasia C - Distemper encephalitis D - Normal MRI E - Chiari malformation

Cerebellar Hypoplasia This puppy most likely has cerebellar hypoplasia, although cerebellar abiotrophy is also possible. Representative T1 and T2 weighted MRI images are presented. The cerebellum is small with a resulting large collection of CSF in the vicinity. There are no other abnormalities of signal intensity. Comparison with normal MRI: http://mirc.veterinaryradiology.net/storage/ss1/docs/20090116144645942/MIRCdocument.xml

Several ornamental freshwater fish (Betta splendens) have recently developed a velvety, rust-colored appearance, especially around the eyes and gills (see below). What is the appropriate treatment in pet fish for the presumptive diagnosis? A - Tricaine&nbsp;methanesulfonate (MS222) B - Chloroquine C - Hydrogen peroxide D - Potassium permanganate E - None; this condition is self-limiting

Chloroquine. This presentation is consistent with "Velvet disease," a parasitic dinoflagellate infection by Piscinoodinium spp. in freshwater fish and Amyloodinium spp. in saltwater species. The treatment in ornamental fish is typically chloroquine or copper sulfate, given as a bath or in a quarantine tank. The Tx in fish raised for food in the U.S. is repeated treatments of copper sulfate (similar to Ich, Ichthyophthirius multifiliis). "Velvet disease" is one of the biggest health concerns in captive marine fish, especially clownfish, and can look like white spots. This nonmotile, photosynthetic, parasitic algae attaches to the skin and gills. There, it invades the tissues and gives rise to cysts that contain flagellated, free-swimming stages of the parasite that go on to initiate new infections. The prognosis is guarded because this disease is difficult to manage. Potassium permanganate and hydrogen peroxide are treatments for oomycete infections in fish (primarily fresh-water). Clinical presentation includes gray-white, cotton-like growths on the skin, eyes, gills, or fins. Tricaine methanesulfonate (MS222, Tricaine mesylate) is the only anesthetic licensed in the United States and Canada for food fish intended for human consumption.

Over the past two weeks, several pigs in a herd have been febrile and depressed. Many were constipated, then had diarrhea. A few were incoordinated and one had seizures. A few have died. Necropsy revealed petechial hemorrhages on the kidneys and larynx, and a hemorrhagic urinary bladder. Of the following choices, which one is most consistent with a presumptive diagnosis? A - Streptococcus suis infection B - Classical swine fever C - Erysipelas D - Glasser's disease E - Swine dysentery

Classical Swine Fever (CSF, "Hog Cholera") Virology is required to confirm and differentiate from other causes of febrile hemorrhagic diseases. The best tissues to submit are the tonsils, maxillary and submandibular lymph nodes, mesenteric lymph nodes, spleen, ileum, and kidney Glaesserella parasuis (Glasser's Disease) is an acute bacterial infection, characterized by different combinations of meningeoencephalitis, polyserositis and polyarthritis and can contribute to bacterial pneumonia.

When a foal is being treated with erythromycin (for Rhodococcus equi, for example), the mare is at risk of developing enterocolitis due to which one of the following organisms? A - Clostridioides difficile (formerly Clostridium difficile) B - Clostridium novyi C - Lawsonia intracellularis&nbsp;(a.k.a. proliferative enteropathy) D - Rhodococcus equi E - Escherichia coli

Clostridioides difficile A history of recent antimicrobial therapy is common in cases of C. difficile associated diarrhea. Adult horses exposed to erythromycin are particularly at risk for C. difficile enterocolitis. C. novyi is the cause of infectious necrotic hepatitis, which is primarily seen in sheep but can also be seen in cattle, hogs and horses. Lawsonia intracellularis is the cause of proliferative enteropathy, resulting in diarrhea and hypoproteinemia in foals and swine. Rhodococcus equi is a notable cause of pneumonia in older foals characterized by pulmonary abscessation as well as some extrapulmonary manifestations. E. coli can be a cause of septicemia and diarrhea in foals and calves.

A two-year-old Angus cow is presented with weakness of the hind limbs and a staggering gait. On physical exam, the heart rate and respiratory rate are slow and irregular, and the pupils are dilated. The cow's breath and urine smell like the odor of mouse urine. Suddenly the cow's pulse becomes rapid and thready, she collapses, and dies of apparent respiratory failure. Which one of the following plants is most likely to have caused this spectrum of clinical signs? A - Veratrum spp (False hellebore, Skunk cabbage) B - Centaurea spp (Knapweed, Yellow star thistle) C - Tetradymia spp (Horsebrush) D - Pinus ponderosa (Western yellow pine) E - Conium maculatum (Poison hemlock)

Conium maculatum (Poison hemlock) Poisoning with Poison hemlock is responsible for the death of this cow. At least eight toxic piperidine alkaloids have been isolated from poison hemlock. Coniine is found in seeds and the mature plants; g-goniceine is found in young growing plants. Poison hemlock is toxic to all livestock and humans. Signs of toxicity develop one to two hours after ingestion and are usually fatal. Signs include nervousness, trembling, weakness (esp. hindlimbs), weak pulse, irregular heart rate, recumbency, coma and death. A mousy odor exuding from the urine and breath is pathognomonic. Ingestion of poison hemlock during gestation causes arthrogryposis and other congenital defects in cattle, goats, and pigs.

A 12yro MN West Highland White Terrier is presented after he disappeared from home for 6hrs and returned with left hindlimb lameness. What is the most appropriate initial treatment? A. Non-steroidal anti-inflammatory medication and rest B. Triple Pelvic osteotomy C. Tibial tubercle transposition and Robert-Jones bandage D. Closed reduction and Ehmer sling E. Femoral head and neck excision

Conservatively treat coxofemoral luxation with closed reduction and an Ehmer sling. Typically presents as a non-weight bearing lameness, usually after blunt force trauma (e.g. Hit by car). Craniodorsal luxation of the femoral head is most common. Tx: Closed or open reduction. Closed reduction and Ehmer sling if injury is < 48hr old, patient is stable for anesthesia, and the hip structure is normal. → 50% success rate. Open reduction if closed reduction is unstable or has failed Carefully monitor post-reduction for irritation/wounds and ischemia caused by Ehmer sling.

A five-year-old Holstein cow is presented with a two-day history of being off feed and a precipitous drop in milk production. The cow stands with abducted elbows, an arched back, and is reluctant to move. Physical exam shows intermandibular edema and bilateral jugular distention. A grunt is heard when pressure is applied to her xiphoid and there is a washing machine murmur (almost like splashing sounds) on both sides. There is little rumen activity. Click the labwork icon to review test results. What is the recommendation for the farmer? Value Normal T=103F (39.4 C) 100.4â€"102.8 F (38.0â€"39.3 C) HR=132 bpm 48-84 bpm BR=36 brpm 26-50 brpm A - Cull this cow B - Test the herd for bovine leukosis C - Treat with high-dose penicillin/streptomycin D - Check the feed for cottonseed meal (gossypol) E - Evaluate the mineral for excess monensin/lasalocid

Cull this cow. Treatment is unrewarding for traumatic reticuloperitonitis. Bilateral dilated jugular veins, a washing machine murmur, the painful stance, and respirator grunting all point to hardware disease (traumatic reticuloperitonitis). Prevent this problem by making every cow swallow a small bar magnet to attract and hold nails, wire, and sharp metal inside the reticulum. A compass can tell you if a cow already has a magnet. Hold it near the brisket and if there is a magnet, the compass needle will point to the cow even if it's moved.

For the last ten years, your state has had a mandatory vaccination program against "pedunculated goblet disease" in fur-bearing turtles and the prevalence of this terrible disease has decreased markedly. How does this decrease in prevalence affect the predictive value positive (PVP) of the best serological test for pedunculated giblet disease? A) PVP is affected by specificity, not prevalence B) PVP depends on the same number tested, not prevalence C) PVP stays the same as prevalence decreases D) PVP increases as prevalence decreases E) PVP decreases as prevalence decreases

E As the disease becomes more and more rare, the predictive value of your same test gets worse. Therefore, as prevalence of a disease goes down, the PVP of your serological test also goes down.

An outbreak of diarrheal disease of piglets has occurred which affected the healthiest animals in the herd, one to two weeks after weaning. Some affected piglets had no signs except peracute death. Other affected piglets exhibit diarrhea, ataxia, paralysis, and recumbency. What condition is at the top of the differential diagnosis list? Epidemic transmissible gastroenteritis (TGE) - Epidemic transmissible gastroenteritis (TGE) Streptococcus suis - Streptococcus suis Clostridium perfringens type C enteritis - Clostridium perfringens type C enteritis Edema disease - Edema disease Porcine proliferative enteritis - Porcine proliferative enteritis

Edema disease is caused by shiga toxin-producing E. coli (STEC). Look for severe acute illness ranging from peracute death with no signs to CNS involvement with ataxia, paralysis, and recumbency in healthiest pigs one to two weeks after weaning. Streptococcus suis could cause sudden death but would not cause diarrhea and would see polyserositis on necropsy. Clostridium perfringens type C enteritis, also called enterotoxemia in other animals, is characterized by a HEMORRHAGIC diarrhea in one- to three-day-old piglets. Porcine proliferative enteritis is principally a diarrheal disease of growing finishing (40- to 80-lb) pigs and young breeding pigs. Epidemic transmissible gastroenteritis (TGE) in non-immune pig herds characterized by HIGH MORBIDITY and HIGH MORTALITY in piglets less than one week old.

A 15-year-old Quarter horse mare is presented for right front lameness grade 3/5 on the AAEP lameness scale (lameness consistently visible at the trot). She becomes sound after an abaxial nerve block. Radiographs of the distal limb look like this: What is the top differential diagnosis? A - Pedal osteitis B - High ringbone C - Spavin D - Chronic proliferative synovitis E - Exostosis of the second metacarpal https://aaep.org/horsehealth/lameness-exams-evaluating-lame-horse?utm_source=Zukureview+Subscribers&utm_campaign=223c511cd8-EMAIL_CAMPAIGN_2022_07_20_01_50&utm_medium=email&utm_term=0_1c9568dbdc-223c511cd8-7312275

High Ringbone (periostitis and osteoarthritis of the proximal interphalangeal joint leading to exostoses) Very common in horses. Can be due to chronic wear and tear, overuse, or secondary to a traumatic episode. Osselets are traumatic athritides of the metacarophalangeal joints. Pedal osteitis is demineralization of the solar margin of the distal phalanx, usually due to inflammation. Bone spavin is osteoarthritis of the distal intertarsal, tarsometatarsal, and less commonly, the proximal intertarsal joints.

A flock from a turkey farm is presented with a mysterious illness. Several dead birds are notable, mostly younger animals. Sick turkeys are listless, with drooping wings, unkempt feathers, yellow droppings. Sick older birds are emaciated. Necropsy shows a yellowish green, caseous exudate in the ceca, cecal ulcerations and thickening of the cecal wall. A - Hemorrhagic enteritis of turkeys B - Necrotic enteritis C - Coronaviral enteritis of turkeys D - Histomoniasis E - Avian spirochetosis

Histomoniaisis: the combination of characteristic "bull's eye" lesions on liver and cecal changes are pathognomonic. Caused by protozoan Histomonas meleagridis, transmitted in eggs of cecal nematode Heterakis gallinarum. Expect a depression/diarrhea presentation. Expect to see more sudden death with necrotic enteritis caused by C. perfringens. Signs of avian spirochetosis are highly variable, may be absent: see listlessness, shivering, increased thirst, green/yellow diarrhea with increased urates early on. Caused by a tick-borne borrelia. Look for characteristic enlarged, mottle spleen with petechial hemorrhages, similar to Marble spleen disease of pheasant.s Expect depression, bloody droppings, substantial mortality with hemorrhagic enteritis of turkeys. Expect diarrheal presentation with coronaviral enteritis of turkeys, but NOT the characteristic cecal/liver lesions described on necrosis from above.

Two eight-week old calves are presented down and extremely weak. They are depressed and lying in pools of foul-smelling brown diarrhea with a small amount of blood. Another calf died suddenly the previous night with no prior signs. The down calves are dehydrated, with rectal temperatures of 105.2° and 105.6° F (40.7° - 40.9° C) [N=101.5°-103.5°F, N=37.8°-39.7° C], respectively. What is the treatment plan? A - Cull sick calves; prophylactic oxytetracycline in feed for well animals B - Isolate sick calves; oral electrolytes C - Immunize calves and adult cattle with MLV vaccine; probiotics for sick calves D - IV fluids; NSAIDS; frequent milk feedings E - Corticosteroids; amprolium in water; rumensin in feed

IV fluids; NSAIDS; frequent milk feedings Fever, diarrhea and sudden death in 8 wk calves is highly suggestive of septicemia due to salmonellosis. Isolate sick calves Use of antibiotics is controversial as they may prolong recovery and shedding and yield a carrier calf. However, if an animal is septic it needs parenteral antibiotics. Prognosis is poor with neonatal salmonella and deaths can approach 100% of affected calves. In adults, antibiotics may yield a clinical cure, but Salmonellae can establish in the biliary system and intermittently shed into the GI system, leading to environmental contamination. Prevention is dependent on which species of Salmonella is causing the problems-host adapted or environmental.

Which one of the following choices includes the cardinal sign of trigeminal neuritis? Inability to close the mouth - Inability to close the mouth Dysphagia, dysphonia and stridor - Dysphagia, dysphonia and stridor Circling and head tilt toward side of lesion, no other signs - Circling and head tilt toward side of lesion, no other signs Masseter muscle pain associated with chewing - Masseter muscle pain associated with chewing Paralyzed eyelid, ear or lip on one or both sides of the face - Paralyzed eyelid, ear or lip on one or both sides of the face

Idiopathic trigeminal neuritis, due to inflammation of cranial nerve 5 (CN 5), is characterized by acute onset of flaccid jaw paralysis. Affected animals cannot close their mouth and have difficulty eating and drinking. Seen occasionally in dogs, rare in cats. Cause is unknown. Idiopathic facial nerve paralysis, affecting cranial nerve 7 (CN 7), results in the inability to move the eyelid, lip, or ear and dryness of the eyes and mouth. Masticatory myositis is characterized by pain on opening the mouth and swelling of the muscles of mastication (acute) or atrophy of the temporalis and masseter muscles with the inability to open the mouth due to fibrosis (chronic). Dysphagia, dsyphonia, and stridor are most often associated with dysfunction of the vagus nerve, cranial nerve 10 (CN 10). Circling and head tilt toward the side of the lesion with no other signs is a common presentation of vestibulocochlear nerve, cranial nerve 8 (CN 8) lesions. Concurrent CN 7 paralysis and Horner's syndrome (ptosis, miosis, enophthalmos) may be present with middle- and inner-ear infections.

A large male guinea pig from a group of four is presented with swollen and scabby hind feet from which Staphylococcus aureus is cultured. Which one of the following recommendations is the most appropriate choice? A - Cull affected animal, increase ventilation of environment for remaining animals B - Improve sanitation, install smooth-floored enclosure C - Isolate affected animal, treat with oral amoxicillin D - Tetracycline-medicated water for all animals E - Debridement and topical 1% butenafine cream

Improve sanitation, install smooth-floored enclosure Bumblefoot (pododermatitis) in guinea pigs usually occurs secondary to poor sanitation, obesity and wire cage floors or rough bedding. If detected early, switch to smooth-bottom flooring, keep the enclosure clean, and change to a softer bedding. Chlorhexidine soaks and debridement can help the feet. Prognosis is guarded. Avoid penicillins in guinea pigs. Guinea pigs are very sensitive to ABX and penicillin can upset the normal GI bacterial flora and will often lead to death.

Which of the following pairs of tissues can both cause increased serum alanine aminotransferase (ALT) levels in dogs and cats? A - Spleen and adrenals B - Kidneys and pancreas C - Liver and muscle D - Heart and kidneys E - Pancreas and intestine

Liver and muscle Damage to the liver and muscle cells of dogs and cats causes increased drum levels of alanine aminotransferase (ALT) ALT is considered a "leakage" enzyme. High levels are normally found in the cytoplasm of healthy cells. When hepatic or muscle cells are damaged, ALT leaks into adjacent tissue where it's picked up by the venous circulation. Other animals, such as horses, ruminants, pigs and birds do not have high levels of ALT inside cells. Aspartate aminotransferase (AST) is the predominant leakage enzyme in these animals. The magnitude of the elevation of leakage enzymes can be deceiving - are a few cells leaking a lot or are many cells leaking a small amount? Severe damage to a healthy liver may result in very high levels (prognosis good), while low levels may be seen when significant atrophy or fibrosis of the liver is present and few cell are left (prognosis grim) Differentiation between muscle and liver as the source of increases in ALT/AST is determined by evaluation of other muscle (creatine phosphokinase and AST) and liver enzymes (AST, sorbitol dehydrogenase (SDH), gamma glutamyl transferase (GGT), alkaline phosphatase (ALP)), the animal, and the clinical evaluation. Leakage enzymes do not provide an estimate of actual liver function. This is evaluated by measurement of substances dependent on the efficacy of liver function such as bile acids, blood urea nitrogen, albumin, and glucose.

A nine-year-old Irish setter is presented for vomiting. The owner states that the dog does not really retch, but just spits up food soon after eating. The dog is normal otherwise. Based on the lateral radiograph below, which one of the following choices is the most likely diagnosis? A - Gastric foreign body B - Pulmonary bullae C - Mediastinal mass D - Megaesophagus E - Persistent right aortic arch

Megaeophagus It can be idiopathic or secondary to several disorders. It is frequent. It's a frequent finding with myasthenia gravis. Other causes of megaesophagus include esophagitis, esophageal stricture or diverticulum, heavy metal toxicity, poly myositis, lupus, hypoadrenocorticism. A thorough neurologic examination, complete blood work including adrenal testing, and possibly contrast tidies of the esophagus should be performed in this patient. Regurgitation is actually present in this dog. This occurs very soon after eating while vomiting usually occurs a few hours later.

A nine-year-old Irish setter is presented for vomiting. The owner states that the dog does not really retch, but just spits up food soon after eating. The dog is normal otherwise. Based on the lateral radiograph below, which one of the following choices is the most likely diagnosis? Image Persistent right aortic arch - Persistent right aortic arch Gastric foreign body - Gastric foreign body Megaesophagus - Megaesophagus Mediastinal mass - Mediastinal mass Pulmonary bullae - Pulmonary bullae

Megaesophagus is present on this radiograph. It can be idiopathic or secondary to several disorders. It is a frequent finding with myasthenia gravis. Other causes of megaesophagus include esophagitis, esophageal stricture or diverticulum, heavy metal toxicity, polymyositis, lupus, hypoadrenocorticism. A thorough neurologic examination, complete bloodwork including adrenal testing, and possibly contrast studies of the esophagus should be performed in this patient. Regurgitation is actually present in this dog. This occurs very soon after eating, while vomiting usually occurs a few hours later. Attention to details of the history is important to help direct diagnostics.

During examination of a newborn male cria, adhesions between the free end of the penis and the prepuce are evident. What is the most likely explanation? A - Common congenital malformation of preputial ring, associated with phimosis B - Normal finding C - Failure of separation of preputial diverticulum D - Sequella of hypospadias; Heritable E - Secondary to stress in utero, 5th month

Normal finding Normal male cries are born with adhesions between the free end of the penis and the prepuce. As they sexually mature these adhesions gradually detach, so puberty can be seen clinically when the penis can be completely exteriorized. Castration prior to puberty may result in continued adhesions. Male llamas reach puberty at approximately 21 months. Alpacas mature earlier around 12 months of age.

For the last ten years, your state has had a mandatory vaccination program against "pedunculated giblet disease" in fur-bearing turtles and the prevalence of this terrible disease has decreased markedly. How does this decrease in prevalence affect the predictive value positive (PVP) of the best serologic test for pedunculated giblet disease? A - PVP increases as prevalence decreases B - PVP depends on the number tested, not prevalence C - PVP stays the same as prevalence decreases D - PVP is affected by specificity, not prevalence E - PVP decreases as prevalence decreases

PVP decreases as prevalence decreases. That is, as your disease becomes more and more rare, the predictive value of your same old test gets worse.

Which nerve block would most specifically relieve lameness resulting from fracture of the navicular bone? A - Tibial and peroneal B - Abaxial sesamoidean C - Palmar digital D - Low four-point E - Median and ulnar

Palmar Digital The palmar digital nerve block (also called the PD or heel block) would anesthetize at least the palmar third of the foot, including the navicular bone. All of the other nerve blocks can desensitize the navicular bone too, but would not be as specific as a palmar digital nerve block. The abaxial sesamoidean nerve block (ASNB, also called a pastern or foot block) would anesthetize the entire foot and much of the pastern. Sometimes an ASNB is needed to completely eliminate or diagnose navicular lameness, but it is less specific than the PD block. A low four-point nerve block (also called low palmar or volar block) would desensitize the fetlock and areas distal to it. The median and ulnar nerve block would anesthetize the carpus and areas distal to it. The tibial and peroneal nerve block would anesthetize the tarsus and areas distal to it. Forelimb landmarks: https://www.merckvetmanual.com/musculoskeletal-system/lameness-in-horses/regional-anesthesia-in-equine-lameness Pelvic limb landmarks: https://www.merckvetmanual.com/musculoskeletal-system/lameness-in-horses/regional-anesthesia-in-equine-lameness

A one-year-old female spayed German Shepherd is presented for a two-week history of lameness. Although the dog limps on the right foreleg as it enters the exam room, the owner reports that the dog was lame on the left hindleg last week. On physical exam, the dog reacts painfully on palpation of the long bones of the right foreleg. What is the most likely diagnosis? Value - Normal RI 99.8°F (37.7°C) - RI: 99.5-102.5°F (37.2-39.2°C) HR=100 bpm - RI: 80-120 RR=24 brpm - RI: 15-34 A - Hypertrophic osteopathy B - Panosteitis C - Osteosarcoma D - Multiple cartilaginous exostoses E - Hypertrophic osteodystrophy

Panosteitis Panosteitis is an acute-onset shifting leg lameness with long bone pain most commonly seen in young (5-18 mo) medium to large breed dogs. German shepherds are at the highest risk of developing Panosteitis. Hypertrophic osteodystrophy (HOD) is an uncommon developmental orthopedic disease of young (2-8mo) growing dogs. Large and giant breeds, in particular the Great Dane, are predisposed. It is characterized by symmetrical distal metaphyseal pain and swelling of long bones (most commonly the ulna and tibia), fever, anorexia, and depression. Hypertrophic osteopathy is a diffuse periosteal proliferation disease of long bones in dogs secondary to a neoplastic mass or other disease (e.g. Heartworm disease) in the thoracic or abdominal cavity. Multiple cartilaginous exostoses is a rare benign proliferation of bone/cartilage that can affect the metaphyseal cortical surfaces of long bones, vertebrae, and ribs in young dogs and cats. Animals may have no signs at all. Osteosarcoma is unlikely in a young dog and does not cause shifting leg lameness.

These larvae shown below have been removed from the topline of beef cattle in the early winter. Which one of the following treatments is the most appropriate in both lactating dairy and beef cattle? A - Rapid hand expulsion/squeezing B - Chloramphenicol C - None. These are self-limiting D - Organophosphate spray E - Pour-on moxidectin

Pour-On Moxidectin These are the larvae of Hypodrma bovis or H. lineatum (the common and northern cattle grub, respectively). Pour on moxidectin or eprinomectin are approved for use in both lactating dairy and beef cattle. Other products are approved for non-lactating beef cattle. Withdrawal times must be followed. Organophosphates are not approved for this use in cattle in the United States. While the grubs can be manually expulsed/squeezed out, care must be taken to prevent rupture of the grubs. Crushed larvae release Antigens leading to a systemic anaphylactic reaction in cattle. These pests are less common now since macrocyclic lactones gained widespread use in the last few decades. The adult flies (gad flies are heel flies) lay eggs on the lower limbs of cattle over the summer. The larvae penetrate the skin and migrate to their respective preferred locations. By the early winter, the larvae arrive in the SQ tissues of the back and make breathing pores. This cyst/warble stage lasts 4-8 wks, culminating in 3rd stage larvae dropping to the ground and completing the life cycle. Clinical presentation of the larval infections range from localized tissue trauma around the warbles/cysts to periostitis, osteomyelitis, and neurological dysfunction. Death of 1st-stage larvae in the esophagus or spinal canal can lead to location-specific clinical signs. Cattle bothered by adult flies in summer show "gadding behavior" in stamping and running with tails in the air.

A commercial swine operation in Iowa reports an outbreak in October among weanling and grower-finisher pigs with 25 dead animals and 54 more exhibiting acute onset of dyspnea and weakness. Twenty-four hours after the first pigs showed clinical signs, some of the worst affected animals became recumbent and died. Physical exam of affected pigs shows cyanosis of the mucous membranes and weakness. Necropsy of a dead animal shows marked pulmonary edema and hydrothorax. The farm is a farrow to finish operation, with same-age animals penned and fed together. The ration for the affected pens is a mixture of corn, rolled oats, dried whey, soybean meal and a vitamin/mineral pack. Weanlings get water from nipple waterers and older animals drink from stainless steel finishing cup waterers, one per pen. The water supply comes from an unchlorinated well that supplies the whole farm. Barns have concrete floors and feature hybrid ventilation with wall-mounted variable speed fans and moveable insulated sidewall curtains, roof-mounted ventilation chimneys, and fresh air intakes in the ceilings. What is the best action to take? A - Remove the feed; Submit samples for mycotoxin testing B - Culture nasopharynx of dead pigs; Isolate and Tx affected pigs with tylosin pending culture/sensitivity results C - Report outbreak to state/provincial veterinarian; Cull affected animals D - Check feces for embryonated eggs; Remove access to soil/earthworms; Treat survivors with ivermectin E - Check humidity, air exchange rates, and positive/negative pressure of affected barns

Remove the feed; submit samples for mycotoxin testing. The most likely differential in this scenario is fumonisin toxicosis (porcine pulmonary edema, PPE); therefore, remove the feed and submit samples for mycotoxin testing. High morbidity and mortality in pigs with respiratory signs and pulmonary edema on necropsy are hallmarks of this disease. PPE appears to be caused by pulmonary hypertension with transudation of fluids into thorax, which leads to interstitial pulmonary edema and hydrothorax. Fumonisin is a Fusarium spp. mycotoxin associated with moldy corn. Other mycotoxins cause other syndromes. Think of reproductive dysfunction (estrogenism, vulvovaginitis) with zearalenone, the only known mycotoxin with primarily estrogenic effects. Trichothecenes are a group of related cytotoxic mycotoxins associated with many fungi. Think of vomitoxin (and vomiting) and also of immunosuppression. Refusal to eat contaminated feed is a typical sign, due to taste aversion. Macrocyclic trichothecene-related diseases have several specific names, including the best known, stachybotryotoxicosis. The primary differential for pulmonary edema is ionophores toxicity. There is no ionophore in the ration for this case scenario. Lungworms (metastrongylus spp.) might cause signs of respiratory disease but require access to earthworms, the intermediate host, so this should not be an issue in swine housed on concrete. Ventilation problems or problems with infectious lung diseases are likely to have a more chronic course with lower morbidity and mortality.

Which one of the following management recommendations is most helpful to reduce the incidence of mare reproductive loss syndrome? A - Remove the horses from affected pastures B - Cut down all the black walnut trees on premises C - Vaccinate mares and stallions against equine coital exanthema D - Progesterone supplementation between four to six months of pregnancy E - Spray paddocks to kill Habronema muscae larvae

Remove the horses from affected pastures. Removal of mares from affected pastures is necessary for control of mare reproductive loss syndrome (MRLS). The exact pathogenesis of MRLS is unknown, but exposure to/ingestion of "hirsute" or hairy caterpillars appears to be a risk factor. Early and late-term abortions have been reproduced by oral administration of whole Eastern tent caterpillars (Malacosoma americanum) or their exoskeletons. Eastern tent caterpillars were associated with a large outbreak in Kentucky in 2001-2 where over 3000 pregnant mares aborted. The most current theory - damage to the oral mucosa or intestinal lining by the setae (spines) of the caterpillars introduces bacteria that causes bacteremia and subsequent infection of the fetus and/or placenta. Early embryonic loss, abortion, stillbriths, weak foals, and infected foals are seen. Swollen umbilical cords are also often seen in aborted fetuses. Feeding hay and removal of wild cherry trees, a major food source for the caterpillars, is also beneficial in reducing the incidence of MRLS abortion.

Some farmed salmon appear lethargic with darkened skin and exophthalmia. Necropsy of one that died reveals vent hemorrhages and grayish granulomatous nodules on the kidneys. Gram stain from the kidney lesion reveals the intracellular bacteria shown below. How could this condition have been prevented? A - Oxytetracycline in water; Cull fish older than 2 years B - Treat females with erythromycin before spawning C - Vaccinate all new arrivals D - Test incoming fish with ELISA and use formalin for any positives E - Increase water temp in tank/pond to 52-63 ºF (11-17°C); Supplement diet with vitamin C

Renibacterium salmoninarum (bacterial kidney disease) is prevented by treating females with erythromycin before spawning to prevent vertical transmission. R. salmoninarum is one of the few gram-positive organisms that causes disease in fish and it is economically important. Fish appear lethargic with darkened skin and sometimes exophthalmia. There may be grayish granulomatous nodules on the kidneys. It is transmitted both horizontally and vertically and disease is more common in younger fish. Bacterial kidney disease is most common in both farmed and wild salmonids, but also occurs in other fish species. Purchase disease-free stock to avoid introducing this into the system. Erythromycin is not FDA-approved for treatment in farmed salmonids and survivors remain carriers.

A six-year-old hunter-jumper horse is presented with a three-month history of poor performance and intermittent shifting hindleg lameness. There is reluctance to bring the hind limbs forward at a trot and he occasionally drags his hind toes. On physical exam, there is poor muscling of the gluteal muscles and some asymmetry to the croup (rump). The horse shows pain and a reluctance to ventroflex the back when midline pressure is applied. Which one of the following is a top differential diagnosis? A - Suspensory desmitis B - Overlapping vertebral spinous processes C - Coxofemoral luxation D - Cauda equina (polyneuritis equi) E - Sacroiliac disease

Sacroiliac Disease Think of sacroiliac (SI) disease in an athletic jumping horse with: - intermittent hindlimb lameness localized in the croup (rump) - evidence of back pain - swelling over the tuber sacrale ("hunter's bumps") Horses with hunter's bumps can be sound if the injury has completely healed, but lameness can be observed if the injury is recent or has never been resolved. It is important to rule out other causes of hind end lameness or back pain/strain. US per rectum or local anesthesia infiltration of the SI region can help confirm the Dx. Most often, treatment is administered instead and the response to therapy is monitored for Dx confirmation. Coxofemoral luxation is rare in horses due to a deep acetabulum and the presence of the accessory ligament of the hip (unique to horses). Overlapping vertebral spinous processes (aka "kissing spines") is a problem of the thoracic and lumbar vertebra under the saddle area, seen in short-backed eventing horses (hunter/jumpers, dressage). Cauda equina (polyneuritis equi) can cause gluteal muscle atrophy, but also causes tail paralysis, urinary and fecal incontinence, perineal analgesia or paresthesia, and mild hind limb ataxia.

A rescued Pekin duck is presented for non-weight bearing lameness of the left pelvic limb and fever. Which one of the following choices is the most likely diagnosis? A - Normal radiographs B - Septic arthritis C - Panosteitis D - Egg binding E - Bumblefoot

Septic arthritis There is diffuse soft tissue swelling of the left pelvic limb. There also appears to be intracapsular soft tissue swelling of the left tibia tarsal joint causing the joint space to widen asymmetrically. An osseous fragment is noted at the medial aspect of the joint. The articular margins of the tibiotarsus and the tarsometatarsus appear lytic and irregular. Egg-binding signs are nonspecific but lameness is not a key finding typically. Sometimes, the bird will adopt a penguin-like posture. Egg-binding is more often seen during the spring and summer.

A female show dog is approaching estrus and the owner has consulted you about planning a breeding for this dog. She is aware of the signs of proestrus and plans to contact you when those are apparent in her dog. In addition to vaginal cytology, which one of the following will be the most useful to you in planning the timing of breeding? A - Determination of serum estrogen levels B - Serial progesterone testing C - Ovarian ultrasonography D - Vaginoscopy E - Timed administration of luteinizing hormone

Serial progesterone testing. Serial progesterone testing is the most commonly used hormonal assay method for estimation of the approximate ovulation day in bitches. Progesterone levels are typically measured every two to three days once signification cornification is seen in the vaginal cytology of a proestrus bitch. Baseline progesterone is 0-1 ng/mL. About 48 hours prior to ovulation, a rise in progesterone (greater than 2ng/mL) is seen, corresponding with the luteinizing hormone (LH) surge, which is the actual stimulus for ovulation. The progesterone level increases to 4-10ng/mL (or in some reports up to 15ng/mL) on ovulation day. For natural breeding or insemination with fresh or chilled semen, dogs are usually bred at least twice, typically on days 2 and 4 after the estimated ovulation date. Serial daily measurements of serum LH is also recognized as a method of estimation of ovulation day in bitches and is in fact the most definitive diagnostic test available. However, LH testing is hampered by expense, necessity for daily testing, and problems with availability of tests. Administration of exogenous LH has not been shown to be safe and effective in inducing estrus in dogs. While vaginoscopy is a useful adjunct in breeding management, progesterone measurement is a more accurate method. Estrogen measurement is of little value for estimation of ovulation day because peak levels vary form dog to dog and do not necessarily correlate with the fertile period. Ovarian ultrasonography is not commonly used in clinical practice in dogs.

During the necropsy of an eight-year-old mixed breed dog from the Southern United States, reactive granulomas in the esophagus containing bright red worms, 40 mm to 70 mm long are found. Which choice is the most likely diagnosis? A - Gastrophilus spp. B - Spirocerca lupi C - Haemonchus placei D - Physaloptera spp. E - Ollulanus tricuspis

Spirocerca lupi A disease of dogs in the Southern U.S. and tropical climates, Spirocerca lupi (esophageal worms) make reactive granule as of variable size in the esophageal, gastric or aortic walls. Spirocerca sis may also lead to aneurysm in the thoracic aorta or an ossifying spondylitis of the posterior thoracic vertebrae. Typically asymptomatic, but large granulomas can cause esophageal obstruction. Large granulomas may become neoplastic (osteosarcoma, fibrosarcoma). Some dogs develop spondylitis or enlargement of the extremities characteristic of hypertrophic osteopathy. All of the other choices are gastric parasites. -Ollulanus tricuspis = uncommon gastric parasite of cats -Physaloptera spp., ostertagia spp., and trichostrongylus spp. = found in the abomasum of ruminants. -Gasterophilus spp. = larvae of horse bot flies, found in the stomach of horses.

A four-year-old Quarter horse is presented with a three-week history of diffuse patchy alopecia on the ventral midline and face, including a "bulls-eye" lesion in the center of the forehead. There is a mild conjunctivitis and uveitis OU (both eyes). A punch skin biopsy looks like the image below. What is the most appropriate treatment? A - Trichlorfon drench, antibiotic ophthalmic ointment TID B - Topical DMSO, oxybendazole every eight weeks C - Do nothing D - Corticosteriods, benzimidazole-piperazine&nbsp;every&nbsp;12 weeks E - Ivermectin now, repeat in four months

The treatment of choice for Onchocerca spp. microfilaria is an avermectin-family drug, with retreatment at four-month intervals. Treated horses show marked improvement. Although adult filaria are not killed by avermectins, in practice most horses are free of disease within 6-12 months. Adult filaria live in calcified nodules in the ligamentum nuchae and produce microfilaria that migrate to the ventral midline, face, neck, and chest. The resulting dermatitis is thought to be a hypersensitivity to microfilarial antigens. Accumulation of microfilaria in the eye may also lead to conjunctivitis and uveitis. Note that Culicoides spp. (no-see-ums, biting midges) carry the onchocerca microfilariae, but do not cause the problems listed above themselves. Instead, biting midges cause sweet itch, a very pruritic warm weather dermatitis that is also secondary to hypersensitivity. Doing nothing is a poor choice in this symptomatic animal. But finding microfilaria in skin biopsies of asymptomatic horses is a common and incidental finding. Because of this, some references debate the role of onchocera in the pathogenesis. It seems likely that variations in immune response play a role in severity of disease presentation. In humans, onchocerciasis is a well-known cause of hypersensitivity-related superficial keratitis (river blindness) and dermatitis.

A 12-week-old chicken from a young backyard flock is presented for evaluation because of weight loss, decreased appetite, and diarrhea. The irises in the affected bird are lighter than normal with irregular pupillary margins, and the hen does not have a normal pupillary light reflex (bottom chicken in image). Several other young birds appear to be growing poorly compared to the rest of the flock. Based on the presumptive diagnosis, how can this problem be prevented going forward? A - Collect pharyngeal swabs from affected birds;&nbsp;Antibiotic Tx for entire flock based on culture and sensitivity B - Cull birds from the same genetic line C - Buy only vaccinated chicks D - Use only mycoplasma-resistant bloodlines for new additions to flock E - Isolate unaffected animals; Breed from survivors of disease

This chicken is likely to have ocular manifestations of Marek disease. There is no treatment, but the problem is preventable when chicks are vaccinated at hatching or before 2 weeks of age. Diffuse infiltration of mononuclear cells can turn the irises of affected chickens from a normal yellow color to a pale tan to gray color ("gray eye"). Cellular infiltration can also occur in many other tissues (skin, nerves, viscera), and there may be palpable tumors in the muscle and skin. Marek disease is common in commercial poultry. Diagnosis is usually based on enlarged nerves and lymphoid tumors in the viscera. Lymphoid leukosis is a key differential. Absence of bursal tumors helps distinguish Marek disease from leukosis. But if you find bursal tumors, Marek disease is still on the DDX list pending other tests, like immunochemistry. Chicks as young as 3 weeks can show Marek disease, but lymphoid leukosis is typically seen in chickens more than 14 weeks old.

A two-year old beagle is presented with patchy alopecia, papules, and thick crusts, on the ventral abdomen, chest, ears, and elbows. The owner reports intense pruritis and a positive pinnal-pedal reflex is observed. A flea comb does not reveal any fleas. Which of the following organisms is depicted in the image?

This is Sarcoptes sacabei, a highly contagious mite. A positive pinnal-pedal reflex (scratching action with the dog's hindlimb when the examiner firmly rubs the edge of the dog's pinna between thumb and forefinger) is seen in 82% of dogs with sarcoptic mange but observed in only 7% of dogs with pruritus caused by other diseases. Cheyletiellosis (walking dandruff) is another highly contagious mite but, unlike Sarcoptes, they have very prominent mouthparts.

A ten-year-old female spayed Persian cat is presented for evaluation of a bleeding lump. There is a two-cm diameter, firm, solitary, hairless, dome-shaped, ulcerated mass on her left neck just behind the jaw that is bleeding. Cytology obtained by fine needle aspirate reveals reveals cohesive clusters of large cells with round to oval nuclei that are tightly adherent to each other (see below). Click the labwork icon to review hematology, blood chemistry and urinalysis test results. Based on these findings, which choice is the most appropriate treatment option? Image A - Immunotherapy, wide margin excision, chemotherapy B - Cryosurgery, debridement, repeat in 3-5 weeks C - Wide excision of&nbsp;mass, remove&nbsp;regional lymph node,&nbsp;prednisolone, radiotherapy D - Debulk, intralesional implant chemotherapy, minimize sun and cigarette smoke exposure E - Surgical removal

This is a basal cell tumor. Basal cells make up the base of the epidermis, and a benign growth of these is called a basal cell tumor or adenoma. Although this tumor is benign, surgical removal is the treatment of choice due to ulceration and secondary inflammation. Basal cell carcinomas (the malignant form) do not form raised masses but appear as ulcers on the head, neck, and legs: surgical removal is also recommended. Basal cell tumors are common in older cats, accounting for 15-26% of all feline skin tumors. Most are slow growing and typically present for months prior to diagnosis. Domestic Longhairs, Himalayans, and Persians are predisposed. Note the characteristic cytology findings of epithelial neoplasia present in this case: cohesive clusters of large, round to polygonal cells, demonstrating distinctive cell borders, that are tightly adherent to each other with round to oval nuclei.

A 15-year-old Quarter horse mare is presented for right front lameness grade 3/5 on the AAEP lameness scale (lameness consistently visible at the trot). She becomes sound after an abaxial nerve block. Radiographs of the distal limb look are shown below. What is the common name for the top differential diagnosis? A - Pedal osteitis B - High ringbone C - Spavin D - Exostosis of the second metacarpal E - Chronic proliferative synovitis

This is an example of high ringbone (i.e., degenerative joint disease [DJD] of the proximal interphalangeal joint). Very common in horses. Can be due to chronic wear and tear, overuse, or secondary to a traumatic episode. Follow this link to see a radiograph of arthrodesis of the proximal interphalangeal joint to resolve DJD. Osselets are traumatic arthritides of the metacarpophalangeal joints. Pedal osteitis is demineralization of the solar margin of the distal phalanx, usually due to inflammation. Bone spavin is osteoarthritis of the distal intertarsal, tarsometarsal, and less commonly, the proximal intertarsal joints.

A four-month-old female mixed breed puppy is presented with a one-month history of generalized wobbliness when walking and head-bobbing when attempting to eat. The signs have not progressed. Two MRI images are shown below. What is the diagnosis? A - Cerebellar hypoplasia B - Lead toxicosis C - Normal MRI D - Distemper encephalitis E - Chiari malformation

This is cerebellar hypoplasia, although cerebellar abiotrophy is also possible. This is a congenital, irreversible condition. Think of kittens with cerebellar hypoplasia due to in utero feline panleukopenia infection. MRI interpretation: Representative T1 and T2 weighted MRI images. The cerebellum is small with a resulting large collection of CSF in the vicinity. There are no other abnormalities of signal intensity. Follow this link and click additional images to see a normal cerebellum in a dog.

A middle-aged, depressed, coughing, exercise-intolerant Doberman pinscher is presented. The dog has a rapid and irregular heart rate. Which initial diagnostic step is most appropriate? A - Bronchoscopy for airway assessment B - Abdominal focused assessment with sonography for triage, trauma, and tracking rapid (AFAST) for hemoabdomen C - Thoracic radiographs to evaluate for congestive heart failure D - Ultrasound for pericardial effusion E - CBC to identify anemia

Thoracic radiographs to evaluate for CHF This is a classic presentation for dilated cardiomyopathy (DCM). Confirmatory tests for diagnosis would be echocardiogram. The prognosis is guarded- with Tx may live 6 mo - 2 years. Px especially poor for Dobies. Large and giant breeds are at an increased risk, including Irish wolfhounds and Great Danes. Boxers can develop arrhythmogenic right ventricular cardiomyopathy, distinct from DCM. Treatment goal is to diminish overt clinical signs and control arrhythmias. Therapy may include diuretics (furosemide, spironolactone), positive inotropes (pimobendan), angiotensin converting enzyme inhibitors (enalapril) and antiarrhythmics (diltiazem, sotalol). Dobermans have two genetic mutations that can lead to development of DCM - PDK4 and DCM2. Each mutation independently can lead to DCM and affected dogs with both are 30x more likely to develop DCM

An intact male whippet presents with an acute onset of swelling and pain in one of his testicles. Which one of the following tests is helpful to diagnose the cause of the swelling before surgery? A - Nuclear scintigraphy scan B - Lateral radiography C - Positive contrast cystography D - Magnetic resonance imaging E - Ultrasonography

Ultrasonography A skilled ultrasonographer can differentiate between testicular torsion, scrotal hernia, or focal lesions in the testicle. This case happened to be a testicular torsion. The testicle on the right is hemorrhagic and infarcted.


Related study sets

SECURITY+ SY0-601 STUDY SET from Mike Myer's Book

View Set

Ataxia in the Stroke Patient quiz 4

View Set

NURS 3101 Basic Care and Comfort Practice EAQ

View Set

Валер'ян Підмогильний "Місто" (питання за змістом)

View Set

B3: Financial Management (Working Capital Management)

View Set

Abnormal Psych Chapter 5- Obsessive-Compulsive and Stressor-Related Disorders

View Set

Section 8, Unit 4: Veterans Administration Loan Guarantee Programs

View Set